We have your learning goals covered with Spring and Summer courses available. Enroll today!

G
Topic
First Poster
Last Poster
k a My Retirement & New Leadership at AoPS
rrusczyk   1571
N Mar 26, 2025 by SmartGroot
I write today to announce my retirement as CEO from Art of Problem Solving. When I founded AoPS 22 years ago, I never imagined that we would reach so many students and families, or that we would find so many channels through which we discover, inspire, and train the great problem solvers of the next generation. I am very proud of all we have accomplished and I’m thankful for the many supporters who provided inspiration and encouragement along the way. I'm particularly grateful to all of the wonderful members of the AoPS Community!

I’m delighted to introduce our new leaders - Ben Kornell and Andrew Sutherland. Ben has extensive experience in education and edtech prior to joining AoPS as my successor as CEO, including starting like I did as a classroom teacher. He has a deep understanding of the value of our work because he’s an AoPS parent! Meanwhile, Andrew and I have common roots as founders of education companies; he launched Quizlet at age 15! His journey from founder to MIT to technology and product leader as our Chief Product Officer traces a pathway many of our students will follow in the years to come.

Thank you again for your support for Art of Problem Solving and we look forward to working with millions more wonderful problem solvers in the years to come.

And special thanks to all of the amazing AoPS team members who have helped build AoPS. We’ve come a long way from here:IMAGE
1571 replies
rrusczyk
Mar 24, 2025
SmartGroot
Mar 26, 2025
k a March Highlights and 2025 AoPS Online Class Information
jlacosta   0
Mar 2, 2025
March is the month for State MATHCOUNTS competitions! Kudos to everyone who participated in their local chapter competitions and best of luck to all going to State! Join us on March 11th for a Math Jam devoted to our favorite Chapter competition problems! Are you interested in training for MATHCOUNTS? Be sure to check out our AMC 8/MATHCOUNTS Basics and Advanced courses.

Are you ready to level up with Olympiad training? Registration is open with early bird pricing available for our WOOT programs: MathWOOT (Levels 1 and 2), CodeWOOT, PhysicsWOOT, and ChemWOOT. What is WOOT? WOOT stands for Worldwide Online Olympiad Training and is a 7-month high school math Olympiad preparation and testing program that brings together many of the best students from around the world to learn Olympiad problem solving skills. Classes begin in September!

Do you have plans this summer? There are so many options to fit your schedule and goals whether attending a summer camp or taking online classes, it can be a great break from the routine of the school year. Check out our summer courses at AoPS Online, or if you want a math or language arts class that doesn’t have homework, but is an enriching summer experience, our AoPS Virtual Campus summer camps may be just the ticket! We are expanding our locations for our AoPS Academies across the country with 15 locations so far and new campuses opening in Saratoga CA, Johns Creek GA, and the Upper West Side NY. Check out this page for summer camp information.

Be sure to mark your calendars for the following events:
[list][*]March 5th (Wednesday), 4:30pm PT/7:30pm ET, HCSSiM Math Jam 2025. Amber Verser, Assistant Director of the Hampshire College Summer Studies in Mathematics, will host an information session about HCSSiM, a summer program for high school students.
[*]March 6th (Thursday), 4:00pm PT/7:00pm ET, Free Webinar on Math Competitions from elementary through high school. Join us for an enlightening session that demystifies the world of math competitions and helps you make informed decisions about your contest journey.
[*]March 11th (Tuesday), 4:30pm PT/7:30pm ET, 2025 MATHCOUNTS Chapter Discussion MATH JAM. AoPS instructors will discuss some of their favorite problems from the MATHCOUNTS Chapter Competition. All are welcome!
[*]March 13th (Thursday), 4:00pm PT/7:00pm ET, Free Webinar about Summer Camps at the Virtual Campus. Transform your summer into an unforgettable learning adventure! From elementary through high school, we offer dynamic summer camps featuring topics in mathematics, language arts, and competition preparation - all designed to fit your schedule and ignite your passion for learning.[/list]
Our full course list for upcoming classes is below:
All classes run 7:30pm-8:45pm ET/4:30pm - 5:45pm PT unless otherwise noted.

Introductory: Grades 5-10

Prealgebra 1 Self-Paced

Prealgebra 1
Sunday, Mar 2 - Jun 22
Friday, Mar 28 - Jul 18
Sunday, Apr 13 - Aug 10
Tuesday, May 13 - Aug 26
Thursday, May 29 - Sep 11
Sunday, Jun 15 - Oct 12
Monday, Jun 30 - Oct 20
Wednesday, Jul 16 - Oct 29

Prealgebra 2 Self-Paced

Prealgebra 2
Tuesday, Mar 25 - Jul 8
Sunday, Apr 13 - Aug 10
Wednesday, May 7 - Aug 20
Monday, Jun 2 - Sep 22
Sunday, Jun 29 - Oct 26
Friday, Jul 25 - Nov 21


Introduction to Algebra A Self-Paced

Introduction to Algebra A
Sunday, Mar 23 - Jul 20
Monday, Apr 7 - Jul 28
Sunday, May 11 - Sep 14 (1:00 - 2:30 pm ET/10:00 - 11:30 am PT)
Wednesday, May 14 - Aug 27
Friday, May 30 - Sep 26
Monday, Jun 2 - Sep 22
Sunday, Jun 15 - Oct 12
Thursday, Jun 26 - Oct 9
Tuesday, Jul 15 - Oct 28

Introduction to Counting & Probability Self-Paced

Introduction to Counting & Probability
Sunday, Mar 16 - Jun 8
Wednesday, Apr 16 - Jul 2
Thursday, May 15 - Jul 31
Sunday, Jun 1 - Aug 24
Thursday, Jun 12 - Aug 28
Wednesday, Jul 9 - Sep 24
Sunday, Jul 27 - Oct 19

Introduction to Number Theory
Monday, Mar 17 - Jun 9
Thursday, Apr 17 - Jul 3
Friday, May 9 - Aug 1
Wednesday, May 21 - Aug 6
Monday, Jun 9 - Aug 25
Sunday, Jun 15 - Sep 14
Tuesday, Jul 15 - Sep 30

Introduction to Algebra B Self-Paced

Introduction to Algebra B
Sunday, Mar 2 - Jun 22
Wednesday, Apr 16 - Jul 30
Tuesday, May 6 - Aug 19
Wednesday, Jun 4 - Sep 17
Sunday, Jun 22 - Oct 19
Friday, Jul 18 - Nov 14

Introduction to Geometry
Tuesday, Mar 4 - Aug 12
Sunday, Mar 23 - Sep 21
Wednesday, Apr 23 - Oct 1
Sunday, May 11 - Nov 9
Tuesday, May 20 - Oct 28
Monday, Jun 16 - Dec 8
Friday, Jun 20 - Jan 9
Sunday, Jun 29 - Jan 11
Monday, Jul 14 - Jan 19

Intermediate: Grades 8-12

Intermediate Algebra
Sunday, Mar 16 - Sep 14
Tuesday, Mar 25 - Sep 2
Monday, Apr 21 - Oct 13
Sunday, Jun 1 - Nov 23
Tuesday, Jun 10 - Nov 18
Wednesday, Jun 25 - Dec 10
Sunday, Jul 13 - Jan 18
Thursday, Jul 24 - Jan 22

Intermediate Counting & Probability
Sunday, Mar 23 - Aug 3
Wednesday, May 21 - Sep 17
Sunday, Jun 22 - Nov 2

Intermediate Number Theory
Friday, Apr 11 - Jun 27
Sunday, Jun 1 - Aug 24
Wednesday, Jun 18 - Sep 3

Precalculus
Sunday, Mar 16 - Aug 24
Wednesday, Apr 9 - Sep 3
Friday, May 16 - Oct 24
Sunday, Jun 1 - Nov 9
Monday, Jun 30 - Dec 8

Advanced: Grades 9-12

Olympiad Geometry
Wednesday, Mar 5 - May 21
Tuesday, Jun 10 - Aug 26

Calculus
Sunday, Mar 30 - Oct 5
Tuesday, May 27 - Nov 11
Wednesday, Jun 25 - Dec 17

Group Theory
Thursday, Jun 12 - Sep 11

Contest Preparation: Grades 6-12

MATHCOUNTS/AMC 8 Basics
Sunday, Mar 23 - Jun 15
Wednesday, Apr 16 - Jul 2
Friday, May 23 - Aug 15
Monday, Jun 2 - Aug 18
Thursday, Jun 12 - Aug 28
Sunday, Jun 22 - Sep 21
Tues & Thurs, Jul 8 - Aug 14 (meets twice a week!)

MATHCOUNTS/AMC 8 Advanced
Friday, Apr 11 - Jun 27
Sunday, May 11 - Aug 10
Tuesday, May 27 - Aug 12
Wednesday, Jun 11 - Aug 27
Sunday, Jun 22 - Sep 21
Tues & Thurs, Jul 8 - Aug 14 (meets twice a week!)

AMC 10 Problem Series
Tuesday, Mar 4 - May 20
Monday, Mar 31 - Jun 23
Friday, May 9 - Aug 1
Sunday, Jun 1 - Aug 24
Thursday, Jun 12 - Aug 28
Tuesday, Jun 17 - Sep 2
Sunday, Jun 22 - Sep 21 (1:00 - 2:30 pm ET/10:00 - 11:30 am PT)
Monday, Jun 23 - Sep 15
Tues & Thurs, Jul 8 - Aug 14 (meets twice a week!)

AMC 10 Final Fives
Sunday, May 11 - Jun 8
Tuesday, May 27 - Jun 17
Monday, Jun 30 - Jul 21

AMC 12 Problem Series
Tuesday, May 27 - Aug 12
Thursday, Jun 12 - Aug 28
Sunday, Jun 22 - Sep 21
Wednesday, Aug 6 - Oct 22

AMC 12 Final Fives
Sunday, May 18 - Jun 15

F=ma Problem Series
Wednesday, Jun 11 - Aug 27

WOOT Programs
Visit the pages linked for full schedule details for each of these programs!


MathWOOT Level 1
MathWOOT Level 2
ChemWOOT
CodeWOOT
PhysicsWOOT

Programming

Introduction to Programming with Python
Monday, Mar 24 - Jun 16
Thursday, May 22 - Aug 7
Sunday, Jun 15 - Sep 14 (1:00 - 2:30 pm ET/10:00 - 11:30 am PT)
Tuesday, Jun 17 - Sep 2
Monday, Jun 30 - Sep 22

Intermediate Programming with Python
Sunday, Jun 1 - Aug 24
Monday, Jun 30 - Sep 22

USACO Bronze Problem Series
Tuesday, May 13 - Jul 29
Sunday, Jun 22 - Sep 1

Physics

Introduction to Physics
Sunday, Mar 30 - Jun 22
Wednesday, May 21 - Aug 6
Sunday, Jun 15 - Sep 14
Monday, Jun 23 - Sep 15

Physics 1: Mechanics
Tuesday, Mar 25 - Sep 2
Thursday, May 22 - Oct 30
Monday, Jun 23 - Dec 15

Relativity
Sat & Sun, Apr 26 - Apr 27 (4:00 - 7:00 pm ET/1:00 - 4:00pm PT)
Mon, Tue, Wed & Thurs, Jun 23 - Jun 26 (meets every day of the week!)
0 replies
jlacosta
Mar 2, 2025
0 replies
k i Adding contests to the Contest Collections
dcouchman   1
N Apr 5, 2023 by v_Enhance
Want to help AoPS remain a valuable Olympiad resource? Help us add contests to AoPS's Contest Collections.

Find instructions and a list of contests to add here: https://artofproblemsolving.com/community/c40244h1064480_contests_to_add
1 reply
dcouchman
Sep 9, 2019
v_Enhance
Apr 5, 2023
k i Zero tolerance
ZetaX   49
N May 4, 2019 by NoDealsHere
Source: Use your common sense! (enough is enough)
Some users don't want to learn, some other simply ignore advises.
But please follow the following guideline:


To make it short: ALWAYS USE YOUR COMMON SENSE IF POSTING!
If you don't have common sense, don't post.


More specifically:

For new threads:


a) Good, meaningful title:
The title has to say what the problem is about in best way possible.
If that title occured already, it's definitely bad. And contest names aren't good either.
That's in fact a requirement for being able to search old problems.

Examples:
Bad titles:
- "Hard"/"Medium"/"Easy" (if you find it so cool how hard/easy it is, tell it in the post and use a title that tells us the problem)
- "Number Theory" (hey guy, guess why this forum's named that way¿ and is it the only such problem on earth¿)
- "Fibonacci" (there are millions of Fibonacci problems out there, all posted and named the same...)
- "Chinese TST 2003" (does this say anything about the problem¿)
Good titles:
- "On divisors of a³+2b³+4c³-6abc"
- "Number of solutions to x²+y²=6z²"
- "Fibonacci numbers are never squares"


b) Use search function:
Before posting a "new" problem spend at least two, better five, minutes to look if this problem was posted before. If it was, don't repost it. If you have anything important to say on topic, post it in one of the older threads.
If the thread is locked cause of this, use search function.

Update (by Amir Hossein). The best way to search for two keywords in AoPS is to input
[code]+"first keyword" +"second keyword"[/code]
so that any post containing both strings "first word" and "second form".


c) Good problem statement:
Some recent really bad post was:
[quote]$lim_{n\to 1}^{+\infty}\frac{1}{n}-lnn$[/quote]
It contains no question and no answer.
If you do this, too, you are on the best way to get your thread deleted. Write everything clearly, define where your variables come from (and define the "natural" numbers if used). Additionally read your post at least twice before submitting. After you sent it, read it again and use the Edit-Button if necessary to correct errors.


For answers to already existing threads:


d) Of any interest and with content:
Don't post things that are more trivial than completely obvious. For example, if the question is to solve $x^{3}+y^{3}=z^{3}$, do not answer with "$x=y=z=0$ is a solution" only. Either you post any kind of proof or at least something unexpected (like "$x=1337, y=481, z=42$ is the smallest solution). Someone that does not see that $x=y=z=0$ is a solution of the above without your post is completely wrong here, this is an IMO-level forum.
Similar, posting "I have solved this problem" but not posting anything else is not welcome; it even looks that you just want to show off what a genius you are.

e) Well written and checked answers:
Like c) for new threads, check your solutions at least twice for mistakes. And after sending, read it again and use the Edit-Button if necessary to correct errors.



To repeat it: ALWAYS USE YOUR COMMON SENSE IF POSTING!


Everything definitely out of range of common sense will be locked or deleted (exept for new users having less than about 42 posts, they are newbies and need/get some time to learn).

The above rules will be applied from next monday (5. march of 2007).
Feel free to discuss on this here.
49 replies
ZetaX
Feb 27, 2007
NoDealsHere
May 4, 2019
Why are there so many Graphs in China TST 2001?
steven_zhang123   0
2 minutes ago
Source: China TST 2001 Quiz 7 P1
Let $k$ be a given integer, $3 < k \leq n$. Consider a graph $G$ with $n$ vertices satisfying the condition: for any two non-adjacent vertices $x$ and $y$ in graph $G$, the sum of their degrees must satisfy $d(x) + d(y) \geq k$. Please answer the following questions and prove your conclusions.

(1) Suppose the length of the longest path in graph $G$ is $l$ satisfying the inequality $3 \leq l < k$, does graph $G$ necessarily contain a cycle of length $l+1$? (The length of a path or cycle refers to the number of edges that make up the path or cycle.)

(2) For the case where $3 < k \leq n-1$ and graph $G$ is connected, can we determine that the length of the longest path in graph $G$, $l \geq k$?

(3) For the case where $3 < k = n-1$, is it necessary for graph $G$ to have a path of length $n-1$ (i.e., a Hamiltonian path)?
0 replies
steven_zhang123
2 minutes ago
0 replies
2025 TST 22
EthanWYX2009   2
N 3 minutes ago by DottedCaculator
Source: 2025 TST 22
Let \( A \) be a set of 2025 positive real numbers. For a subset \( T \subseteq A \), define \( M_T \) as the median of \( T \) when all elements of \( T \) are arranged in increasing order, with the convention that \( M_\emptyset = 0 \). Define
\[
P(A) = \sum_{\substack{T \subseteq A \\ |T| \text{ odd}}} M_T, \quad Q(A) = \sum_{\substack{T \subseteq A \\ |T| \text{ even}}} M_T.
\]Find the smallest real number \( C \) such that for any set \( A \) of 2025 positive real numbers, the following inequality holds:
\[
P(A) - Q(A) \leq C \cdot \max(A),
\]where \(\max(A)\) denotes the largest element in \( A \).
2 replies
EthanWYX2009
Today at 2:50 PM
DottedCaculator
3 minutes ago
The Quest for Remainder
steven_zhang123   0
5 minutes ago
Source: China TST 2001 Quiz 6 P3
Given sets $A = \{1, 4, 5, 6, 7, 9, 11, 16, 17\}$, $B = \{2, 3, 8, 10, 12, 13, 14, 15, 18\}$, if a positive integer leaves a remainder (the smallest non-negative remainder) that belongs to $A$ when divided by 19, then that positive integer is called an $\alpha$ number. If a positive integer leaves a remainder that belongs to $B$ when divided by 19, then that positive integer is called a $\beta$ number.
(1) For what positive integer $n$, among all its positive divisors, are the numbers of $\alpha$ divisors and $\beta$ divisors equal?
(2) For which positive integers $k$, are the numbers of $\alpha$ divisors less than the numbers of $\beta$ divisors? For which positive integers $l$, are the numbers of $\alpha$ divisors greater than the numbers of $\beta$ divisors?
0 replies
steven_zhang123
5 minutes ago
0 replies
Very chaotic
steven_zhang123   0
9 minutes ago
Source: Chinese TST 2001 Quiz 6 P2
Let \( \varphi \) be the Euler's totient function.

1. For any given integer \( a > 1 \), does there exist \( l \in \mathbb{N}_+ \) such that for any \( k \in \mathbb{N}_+ \), \( l \mid k \) and \( a^2 \nmid l \), \( \frac{\varphi(k)}{\varphi(l)} \) is a non-negative power of \( a \)?
2. For integer \( x > a \), are there integers \( k_1 \) and \( k_2 \) satisfying:
\[
   \varphi(k_i) \in \left ( \frac{x}{a} ,x \right ], i = 1,2; \quad \varphi(k_1) \neq \varphi(k_2).
   \]And these two different \( k_i \) correspond to the same \( l_1 \) and \( l_2 \) as described in (1), yet \( \varphi(l_1) = \varphi(l_2) \).
3. Define \( \#E \) as the number of elements in set \( E \). For integer \( x > a \), let \( V(x) = \#\{v \in \mathbb{N}_+ \mid v = \varphi(k) \leq x\} \) and \( W(x) = \#\{w \in \mathbb{N}_+ \mid w = \varphi(l) \leq x, a^2 \mid l\} \). Compare \( V\left( \frac{x}{a} \right) \) with \( W(x) \).
0 replies
steven_zhang123
9 minutes ago
0 replies
Monkeys have bananas
nAalniaOMliO   3
N 11 minutes ago by jkim0656
Source: Belarusian National Olympiad 2025
Ten monkeys have 60 bananas. Each monkey has at least one banana and any two monkeys have different amounts of bananas.
Prove that any six monkeys can distribute their bananas between others such that all 4 remaining monkeys have the same amount of bananas.
3 replies
nAalniaOMliO
Yesterday at 8:20 PM
jkim0656
11 minutes ago
FE f(x)f(y)+1=f(x+y)+f(xy)+xy(x+y-2)
steven_zhang123   0
19 minutes ago
Find all functions $f: \mathbb{R} \rightarrow \mathbb{R}$ such that for all $x,y \in \mathbb{R}$, we have $f(x)f(y)+1=f(x+y)+f(xy)+xy(x+y-2)$.
0 replies
steven_zhang123
19 minutes ago
0 replies
Problem 4
teps   74
N 22 minutes ago by bjump
Find all functions $f:\mathbb Z\rightarrow \mathbb Z$ such that, for all integers $a,b,c$ that satisfy $a+b+c=0$, the following equality holds:
\[f(a)^2+f(b)^2+f(c)^2=2f(a)f(b)+2f(b)f(c)+2f(c)f(a).\]
(Here $\mathbb{Z}$ denotes the set of integers.)

Proposed by Liam Baker, South Africa
74 replies
teps
Jul 11, 2012
bjump
22 minutes ago
Cono Sur Olympiad 2011, Problem 6
Leicich   22
N 23 minutes ago by cosinesine
Let $Q$ be a $(2n+1) \times (2n+1)$ board. Some of its cells are colored black in such a way that every $2 \times 2$ board of $Q$ has at most $2$ black cells. Find the maximum amount of black cells that the board may have.
22 replies
1 viewing
Leicich
Aug 23, 2014
cosinesine
23 minutes ago
Brasil NMO (OBM) - 2007
oscar_sanz012   0
38 minutes ago
Show that there exists an integer ? such that
[tex3]\frac{a^{29} - 1}{a - 1}[/tex3]
have at least 2007 distinct prime factors.
0 replies
1 viewing
oscar_sanz012
38 minutes ago
0 replies
VERY HARD MATH PROBLEM!
slimshadyyy.3.60   4
N 41 minutes ago by slimshadyyy.3.60
Let a ≥b ≥c ≥0 be real numbers such that a^2 +b^2 +c^2 +abc = 4. Prove that
a+b+c+(√a−√c)^2 ≥3.
4 replies
slimshadyyy.3.60
an hour ago
slimshadyyy.3.60
41 minutes ago
Functional Equation!
EthanWYX2009   1
N 44 minutes ago by DottedCaculator
Source: 2025 TST 24
Find all functions $f:\mathbb Z\to\mathbb Z$ such that $f$ is unbounded and
\[2f(m)f(n)-f(n-m)-1\]is a perfect square for all integer $m,n.$
1 reply
EthanWYX2009
Today at 10:48 AM
DottedCaculator
44 minutes ago
Solve this hard problem:
slimshadyyy.3.60   1
N an hour ago by FunnyKoala17
Let a,b,c be positive real numbers such that x +y+z = 3. Prove that
yx^3 +zy^3+xz^3+9xyz≤ 12.
1 reply
slimshadyyy.3.60
an hour ago
FunnyKoala17
an hour ago
Hard geometry
jannatiar   1
N an hour ago by alinazarboland
Source: 2024 AlborzMO P4
In triangle \( ABC \), let \( I \) be the \( A \)-excenter. Points \( X \) and \( Y \) are placed on line \( BC \) such that \( B \) is between \( X \) and \( C \), and \( C \) is between \( Y \) and \( B \). Moreover, \( B \) and \( C \) are the contact points of \( BC \) with the \( A \)-excircle of triangles \( BAY \) and \( AXC \), respectively. Let \( J \) be the \( A \)-excenter of triangle \( AXY \), and let \( H' \) be the reflection of the orthocenter of triangle \( ABC \) with respect to its circumcenter. Prove that \( I \), \( J \), and \( H' \) are collinear.

Proposed by Ali Nazarboland
1 reply
jannatiar
Mar 4, 2025
alinazarboland
an hour ago
IMO ShortList 1998, number theory problem 6
orl   28
N 2 hours ago by Zany9998
Source: IMO ShortList 1998, number theory problem 6
For any positive integer $n$, let $\tau (n)$ denote the number of its positive divisors (including 1 and itself). Determine all positive integers $m$ for which there exists a positive integer $n$ such that $\frac{\tau (n^{2})}{\tau (n)}=m$.
28 replies
orl
Oct 22, 2004
Zany9998
2 hours ago
Circumcircles eat fish
Eyed   23
N Aug 18, 2024 by Ywgh1
Source: 2020 ISL G7
Let $P$ be a point on the circumcircle of acute triangle $ABC$. Let $D,E,F$ be the reflections of $P$ in the $A$-midline, $B$-midline, and $C$-midline. Let $\omega$ be the circumcircle of the triangle formed by the perpendicular bisectors of $AD, BE, CF$.

Show that the circumcircles of $\triangle ADP, \triangle BEP, \triangle CFP,$ and $\omega$ share a common point.
23 replies
Eyed
Jul 20, 2021
Ywgh1
Aug 18, 2024
Circumcircles eat fish
G H J
Source: 2020 ISL G7
The post below has been deleted. Click to close.
This post has been deleted. Click here to see post.
Eyed
1065 posts
#1 • 9 Y
Y by itised, VulcanForge, tree_3, centslordm, Smileyklaws, megarnie, HWenslawski, jhu08, TheHU-1729
Let $P$ be a point on the circumcircle of acute triangle $ABC$. Let $D,E,F$ be the reflections of $P$ in the $A$-midline, $B$-midline, and $C$-midline. Let $\omega$ be the circumcircle of the triangle formed by the perpendicular bisectors of $AD, BE, CF$.

Show that the circumcircles of $\triangle ADP, \triangle BEP, \triangle CFP,$ and $\omega$ share a common point.
This post has been edited 1 time. Last edited by Eyed, Jul 20, 2021, 8:57 PM
Z K Y
The post below has been deleted. Click to close.
This post has been deleted. Click here to see post.
Aryan-23
558 posts
#2 • 3 Y
Y by centslordm, jhu08, bhan2025
Let $\ell_A$ denote the perpendicular bisector of $AD$. Define $\ell_B$ and $\ell_C$ similarly. Define $X= \ell_B \cap \ell_C$. Define $Y,Z$ similarly.


Claim : $\omega_A,\omega_B, \omega_C$ concur at a point $Q\neq P$

Proof : We show that the orthocenter $H$ of $\triangle ABC$ has the same power wrt all the three circles. Let $H_A,H_B,H_C$ denote the feet of altitude from $A,B,C$ to their opposite sidelines. Note that since $\omega_A$ is symmetric in the $A$-midline, $H_A$ must lie on $\omega_A$. Hence the power of $H$ wrt $\omega_A$ is $HH_A\cdot HA$ and similarly for others. Since $HH_A\cdot HA= HH_B \cdot HB = HH_C \cdot HC$, we are done. $\square$

Let $\ell$ denote the line through the centers $O_A$, $O_B$ and $O_C$ of $\omega_A$, $\omega_B$, $\omega_C$ respectively. Note that $Q$ is the reflection of $P$ in $\ell$.


Claim : $Q$ is the miquel point of the quadrilateral formed by the lines $\ell, \ell_A, \ell_B, \ell_C$.

Proof : We prove $Q\in (O_BO_CX)$. Two similar other cyclic quads imply the result. Angle Chase :

$$ \measuredangle O_CQO_B = \measuredangle (O_CQ, \ell) + \measuredangle (QO_B, \ell) = \measuredangle PBQ + \measuredangle QCP = \measuredangle CQB + \measuredangle BPC = \measuredangle PQB + \measuredangle CQP + \measuredangle BAC = \measuredangle H_BEB + \measuredangle FH_CC+\measuredangle (BH_B,CH_C) = \measuredangle (BE,CF) = \measuredangle (\ell_B, \ell_C) = \measuredangle O_CXO_B $$
The above claim implies $Q \in \odot (XYZ)$. $\square$
This post has been edited 1 time. Last edited by Aryan-23, Jul 21, 2021, 12:10 PM
Z K Y
The post below has been deleted. Click to close.
This post has been deleted. Click here to see post.
mathaddiction
308 posts
#5 • 4 Y
Y by centslordm, jhu08, KilleR_1234, MrOreoJuice
Let $H$ be the orthocenter and $O$ be the circumcenter of $\triangle ABC$. Let $H_A,H_B,H_C$ be the altitudes from $A,B,C$ and $M_A,M_B,M_C$ be the midpoints of $BC,CA,AB$.
Notice that $AH_APD$, $BH_BPE$ and $CH_CPF$ are all cyclic being isosceles trapezoids.
Let $Q$ be the point on $PH$ such that $QH\times HP=AH\times HH_A$, then
$$QH\times HP=AH\times HH_A=BH\times HH_B=CH\times HH_C$$Therefore, $Q$ lies on $\omega_A,\omega_B,\omega_C$ by power of a point.
[asy]
      size(10cm); 
real labelscalefactor = 0.5; /* changes label-to-point distance */
pen dps = linewidth(0.7) + fontsize(10); defaultpen(dps); /* default pen style */ 
pen dotstyle = black; /* point style */ 
real xmin = -10.025435123270157, xmax = 9.520748651809164, ymin = -9.136061997969437, ymax = 2.346225331785679;  /* image dimensions */
pen zzttqq = rgb(0.6,0.2,0); pen qqwuqq = rgb(0,0.39215686274509803,0); pen zzttff = rgb(0.6,0.2,1); 
 /* draw figures */
draw(circle((-4.037913311076221,-2.2566115038288466), 2.8107234008805477), linewidth(0.8) + zzttqq); 
draw(circle((-3.8712516863572652,-1.6916842646330805), 2.41040864269716), linewidth(0.8) + linetype("4 4") + qqwuqq); 
draw(circle((-1.5338234255931404,-6.281407059801208), 5.471445122829015), linewidth(0.8) + linetype("4 4") + qqwuqq); 
draw(circle((-3.5874345532208927,-2.2489813536951577), 2.43094335885901), linewidth(0.8) + linetype("4 4") + qqwuqq); 
draw((-1.6275824485579156,-0.8107653261704921)--(-5.903242836377741,-2.9882496471061866), linewidth(0.8) + linetype("4 4") + blue); 
draw((-5.277489523809524,0.266009523809525)--(-6.4,-3.78), linewidth(0.8) + zzttff); 
draw((-6.4,-3.78)--(-1.58,-3.62), linewidth(0.8) + zzttff); 
draw((-1.58,-3.62)--(-5.277489523809524,0.266009523809525), linewidth(0.8) + zzttff); 
draw((-5.838744761904762,-1.7569952380952374)--(-3.428744761904762,-1.6769952380952375), linewidth(0.8)); 
draw((-3.428744761904762,-1.6769952380952375)--(-3.99,-3.7), linewidth(0.8)); 
draw((-3.99,-3.7)--(-5.838744761904762,-1.7569952380952374), linewidth(0.8)); 
 /* dots and labels */
dot((-5.277489523809524,0.266009523809525),dotstyle); 
label("$A$", (-5.1827147036596966,0.4836405550124255), NE * labelscalefactor); 
dot((-6.4,-3.78),dotstyle); 
label("$B$", (-6.322178331803334,-3.570220429729362), NE * labelscalefactor); 
dot((-1.58,-3.62),dotstyle); 
label("$C$", (-1.50137067427256,-3.394918333091879), NE * labelscalefactor); 
dot((-1.6275824485579156,-0.8107653261704921),dotstyle); 
label("$P$", (-1.5451961984319307,-0.590084786892156), NE * labelscalefactor); 
dot((-3.99,-3.7),linewidth(4pt) + dotstyle); 
label("$M_{A}$", (-3.911774503037947,-3.5263949055699912), NE * labelscalefactor); 
dot((-3.428744761904762,-1.6769952380952375),linewidth(4pt) + dotstyle); 
label("$M_{B}$", (-3.3420426889661283,-1.5104207942389403), NE * labelscalefactor); 
dot((-5.838744761904762,-1.7569952380952374),linewidth(4pt) + dotstyle); 
label("$M_{C}$", (-5.752446517731515,-1.5761590804779961), NE * labelscalefactor); 
dot((-1.5741017749957276,-2.4218706172314066),dotstyle); 
label("$D$", (-1.4794579121928746,-2.211629180788871), NE * labelscalefactor); 
dot((-6.993049146010329,-5.915939871582023),dotstyle); 
label("$E$", (-6.913822907954838,-5.695758351458839), NE * labelscalefactor); 
dot((-4.526153724567535,-0.006596031916806355),dotstyle); 
label("$F$", (-4.437680792950395,0.2206874100562014), NE * labelscalefactor); 
dot((-5.181662901657082,-2.6207674685327818),linewidth(4pt) + dotstyle); 
label("$H$", (-5.095063655340955,-2.4526695636654097), NE * labelscalefactor); 
dot((-5.903242836377741,-2.9882496471061866),linewidth(4pt) + dotstyle); 
label("$Q$", (-5.8181848039705715,-2.803273756940375), NE * labelscalefactor); 
dot((-5.144565528772486,-3.738325826681245),linewidth(4pt) + dotstyle); 
label("$H_A$", (-5.051238131181584,-3.570220429729362), NE * labelscalefactor); 
dot((-3.7903512955040726,-1.2969518534176032),linewidth(4pt) + dotstyle); 
label("$H_B$", (-3.692646882241094,-1.1159910768046042), NE * labelscalefactor); 
dot((-6.01429792221089,-2.3897641730846013),linewidth(4pt) + dotstyle); 
label("$H_C$", (-5.927748614368998,-2.211629180788871), NE * labelscalefactor); 
clip((xmin,ymin)--(xmin,ymax)--(xmax,ymax)--(xmax,ymin)--cycle); 
[/asy]
We now show that $Q$ lies on $\omega$ as well which will complete the proof.
Claim. $Q,Z,O_A,O_C$ are concyclic.
Proof.
This is just angle chasing.
\begin{align*}
     \angle QO_AZ&=\angle AO_AZ-\angle AO_AQ\\
     &=180^{\circ}-\angle APD-2\angle APH\\
     &=\angle AHP-\angle APH
 \end{align*}\begin{align*}
     \angle QO_CZ&=\angle FO_CZ-\angle FO_CQ\\&=\angle FPC-2\angle FPQ\\&=\angle QPC-\angle FPQ
 \end{align*}[asy]
       size(10cm); 
real labelscalefactor = 0.5; /* changes label-to-point distance */
pen dps = linewidth(0.7) + fontsize(10); defaultpen(dps); /* default pen style */ 
pen dotstyle = black; /* point style */ 
real xmin = -11.087065689421976, xmax = 0.6710017679701562, ymin = -9.681903556798279, ymax = 2.6405511385486715;  /* image dimensions */
pen zzttqq = rgb(0.6,0.2,0); pen zzttff = rgb(0.6,0.2,1); pen ttzzqq = rgb(0.2,0.6,0); 
 /* draw figures */
draw(circle((-4.037913311076221,-2.2566115038288466), 2.8107234008805477), linewidth(0.8) + zzttqq); 
draw((-1.6898775280504954,-0.7116538999159499)--(-5.893742741384873,-3.0100930250914564), linewidth(0.8) + linetype("4 4") + blue); 
draw((-5.277489523809524,0.266009523809525)--(-6.4,-3.78), linewidth(0.8) + zzttff); 
draw((-6.4,-3.78)--(-1.58,-3.62), linewidth(0.8) + zzttff); 
draw((-1.58,-3.62)--(-5.277489523809524,0.266009523809525), linewidth(0.8) + zzttff); 
draw((-3.884080522551288,-1.6921101181166989)--(-1.160240268822567,-6.674037707108505), linewidth(0.8) + red); 
draw(circle((-6.304981146793863,-3.9320460032951523), 1.009511921723559), linewidth(0.8) + linetype("4 4") + blue); 
draw(circle((-4.770208019710931,-2.532101009100448), 1.220985928730586), linewidth(0.8) + linetype("4 4") + red); 
draw(circle((-3.8569832421468813,-5.268386404975224), 3.041098197586366), linewidth(0.8) + linetype("4 4") + red); 
draw(circle((-3.3799031645392175,-4.652039837831397), 3.002562031662692), linewidth(0.8) + linetype("4 4") + red); 
draw((-3.884080522551288,-1.6921101181166989)--(-6.368668526955157,-4.939546989751335), linewidth(0.8) + ttzzqq); 
draw((-6.858049709277972,-4.77657515335082)--(-1.160240268822567,-6.674037707108505), linewidth(0.8) + ttzzqq); 
draw((-6.858049709277972,-4.77657515335082)--(-3.5848021187049826,-2.2394929298914716), linewidth(0.8) + ttzzqq); 
 /* dots and labels */
dot((-5.277489523809524,0.266009523809525),dotstyle); 
label("$A$", (-5.184515825811127,0.5005828613033043), NE * labelscalefactor); 
dot((-6.4,-3.78),dotstyle); 
label("$B$", (-6.313290301720771,-3.5441923440395877), NE * labelscalefactor); 
dot((-1.58,-3.62),dotstyle); 
label("$C$", (-1.4924826441899974,-3.379579399636098), NE * labelscalefactor); 
dot((-1.6898775280504954,-0.7116538999159499),dotstyle); 
label("$P$", (-1.5865471838491345,-0.4870948051176345), NE * labelscalefactor); 
dot((-1.629686949426607,-2.5248950809605866),dotstyle); 
label("$D$", (-1.539514914019566,-2.2978371935560222), NE * labelscalefactor); 
dot((-7.088942846384126,-5.8487971134333065),dotstyle); 
label("$E$", (-6.995258214249515,-5.613612216540602), NE * labelscalefactor); 
dot((-4.421699276839515,0.04625299922881543),dotstyle); 
label("$F$", (-4.337934968878893,0.288937647070246), NE * labelscalefactor); 
dot((-5.181662901657082,-2.6207674685327818),linewidth(4pt) + dotstyle); 
label("$H$", (-5.09045128615199,-2.4389340030447277), NE * labelscalefactor); 
dot((-5.893742741384873,-3.0100930250914564),linewidth(4pt) + dotstyle); 
label("$Q$", (-5.7959353335955175,-2.815192161681276), NE * labelscalefactor); 
dot((-1.160240268822567,-6.674037707108505),linewidth(4pt) + dotstyle); 
label("$O_B$", (-1.0691922157238807,-6.48370920838762), NE * labelscalefactor); 
dot((-3.884080522551288,-1.6921101181166989),linewidth(4pt) + dotstyle); 
label("$O_A$", (-3.7970638658388554,-1.4982886064533576), NE * labelscalefactor); 
dot((-3.5848021187049826,-2.2394929298914716),linewidth(4pt) + dotstyle); 
label("$O_C$", (-3.49135411194666,-2.06267584440818), NE * labelscalefactor); 
dot((-5.349204909943023,-3.6070750735595207),linewidth(4pt) + dotstyle); 
label("$Z$", (-5.2550642305554796,-3.4266116694656663), NE * labelscalefactor); 
dot((-6.368668526955157,-4.939546989751335),linewidth(4pt) + dotstyle); 
label("$X$", (-6.266258031891203,-4.743515224693585), NE * labelscalefactor); 
dot((-6.858049709277972,-4.77657515335082),linewidth(4pt) + dotstyle); 
label("$Y$", (-6.760096865101673,-4.578902280290095), NE * labelscalefactor); 
clip((xmin,ymin)--(xmin,ymax)--(xmax,ymax)--(xmax,ymin)--cycle); 
[/asy]
Meanwhile
\begin{align*}
     \angle AHP+\angle FPQ&=\angle AHP+\angle PHC\\
     &=\angle AHC\\
     &=180^{\circ}-\angle ABC\\
     &=\angle APC\\
     &=\angle APH+\angle QPC
 \end{align*}Therefore $\angle QO_AZ=\angle QO_CZ$ and we are done. $\blacksquare$
Now by symmetry $Q,Y,O_A,O_C$ are concyclic and $Q,X,O_B,O_A$ are concyclic. Therefore, $Q$ is the miquel point of $ZXQ_BQ_C$, and so $Q,Z,Y,X$ are concyclic as desired.
Therefore $\omega$ passes through $Q$ as well and the proof is thus completed.
Z K Y
The post below has been deleted. Click to close.
This post has been deleted. Click here to see post.
psi241
49 posts
#6 • 4 Y
Y by centslordm, jhu08, Kagebaka, TechnoLenzer
First, we will show that $\omega_A,\omega_B,\omega_C$ (circumcircles of $\triangle ADP,\triangle BEP,\triangle CFP$ respectively) are coaxial. Let $A_1,B_1,C_1$ be foot of altitude from $A,B,C$ respectively.
Since $\square AA_1DP$ is a trapezoid, $A_1$ must lie on $\omega_A$.
Let $Q$ be an image of $P$ by the negative inversion which center at $H$ and sent $A,B,C$ to $A_1,B_1,C_1$. Hence, $AH\cdot HA_1=BH\cdot HB_1=CH\cdot HC_1=PH\cdot HQ$. Therefore, $Q$ lies on $\omega_A,\omega_B,\omega_C$.
The rest is to show that $Q$ lies on $\omega$.
Let $\ell_A,\ell_B,\ell_C$ be perpendicular bisectors of $AD,BE,CF$ respectively, $Q_A,Q_B,Q_C$ be a reflection of $Q$ across $\ell_A,\ell_B,\ell_C$.

Claim: $P$,$Q_A$,$Q_B$,$Q_C$ are collinear.

It is sufficient to prove that $P,Q_A,Q_B$ are collinear, or equivalently, $\measuredangle QQ_AP-\measuredangle QQ_BP=\measuredangle  Q_AQQ_B$. ($Q_C$ can be done similarly.)
In fact, This can be proved by angle chasing.
Let $R$ be an isogonal conjugate of $P$ wrt $\triangle A_1B_1C_1$.
Notice that $QQ_AAD$ is an trapezoid, so $Q_A$ lies on $\omega_A$ and $QQ_A//AD$.
By angle chasing,
\begin{align*}
\measuredangle Q_AQQ_B &=\measuredangle (AD,BE)\\
&=\measuredangle (AD,AA_1)+\measuredangle (AA_1,BB_1)+\measuredangle (BB_1,BE)\\
&=\measuredangle (AA_1,A_1P)+\measuredangle (BC,CA)+\measuredangle (B_1P,BB_1)\\
&=\measuredangle AA_1P+\measuredangle PB_1B+\measuredangle BCA
\end{align*}, and
\begin{align*}
\measuredangle QQ_AP-\measuredangle QQ_BP &=\measuredangle QAP-\measuredangle QBP \\
&=\measuredangle AQP+\measuredangle QPA-\measuredangle BQP -\measuredangle QPB\\
&=\measuredangle AA_1P- \measuredangle BB_1P+\measuredangle BPA\\
&=\measuredangle AA_1P+\measuredangle PB_1B+\measuredangle BCA.
\end{align*}Therefore, $\measuredangle Q_AQQ_B=\measuredangle QQ_AP-\measuredangle QQ_BP$ which means that $P,Q_A,Q_B$ are collinear. Similarly, $P,Q_A,Q_B,Q_C$ lie on a line.
By Simson's line, $Q$ must lie on circumcircle of a triangle formed by $\ell_A,\ell_B,\ell_C$.
Z K Y
The post below has been deleted. Click to close.
This post has been deleted. Click here to see post.
lneis1
243 posts
#7 • 3 Y
Y by centslordm, L567, jhu08
Solution by L567


We will first characterize $D,E,F$ in a nice way that does not involve those weird midpoints. Let $XYZ$ be the orthic triangle.

Claim: $ADPX$ is cyclic

Proof: Note that $X$ and $A$ are reflections across the midline and so are $P,D$. So, $AXDP$ is an isosceles trapezoid and hence cyclic.

So henceforth, we will only look at $D$ as the point on $(AXP)$ such that $PD \perp BC$

Now, to characterize the common point. Let $H$ be the orthocenter of $\triangle ABC$

Claim: Let $PH$ meet the nine point circle of $ABC$ for the second time at $Q$. Then, $Q$ lies on $\omega_A, \omega_B, \omega_C$

Proof: Its enough to show it lies on $\omega_A$ since the others follow similarly.

Let $HX, HQ$ meet $(ABC)$ at $R,S$. Then, since $QX || SR$, ($Q,X$ are midpoints of $HR, HS$ by definition of nine point circle), we have $\angle XQP = \angle RSP = \angle RAP = \angle XAP$ and so $Q \in (APX) = \omega_A$, as desired.

Now, we must show that $Q \in \omega$ as well, which is the hard part of the problem.

Let $O_1, O_2$ be circumcenters of $\omega_A, \omega_C$. Let the perpendicular bisectors of $AD, CF$ meet at $T$.

Claim: $O_1O_2TQ$ is cyclic

Proof: This is going to be a very long chase of angels to show $\angle O_1TO_2 = \angle O_1QO_2$

Let $K$ be an arbitrary point on $O_2O_1$ beyond $O_1$ which I'll use for the angle chase. Its probably not necessary but whatever.

We have $\angle O_1TO_2 = \angle TO_1K - \angle TO_2K$

Now, see that $\angle TO_1K = \angle AO_1T - \angle AO_1K$ and $\angle TO_2K = \angle CO_2K - \angle CO_2T$

So, $\angle TO_1K - \angle TO_2K = \angle AO_1T - \angle CO_2K - \angle AO_1K + \angle CO_2T$

But $\angle AO_1T = \frac{\angle AO_1D}{2} = \angle APD$ and similarly, $\angle CO_2T = 180 - \angle CPF$

So this means $\angle O_1TO_2 = 180 - \angle CPF + \angle APD - \angle AO_1K - \angle CO_2K$

Observe that $\angle APD = 180 - \angle XAP$ and $\angle CPF = \angle PFZ = \angle PCZ$.

So, $\angle APD - \angle CPF = 180 - (\angle XAP + \angle PCZ) = 180 - ((\angle BAP - \angle BAX) + (\angle PCB - \angle BCZ)) = 180 - ((\angle BAP + \angle PCB) - \angle BCZ - \angle BAX) = \angle BCZ + \angle BAX = 180 - 2 \angle ABC$

So, $\angle O_1TO_2 = 360 - 2 \angle ABC - \angle AO_1K - \angle CO_2K$

Now, we need a way to deal with this annoying $AO_1K$ and $CO_2K$

Let $AO_1$ meet $BC$ at $U$, $CO_2$ meet $AB$ at $V$ and $AO_1$ meet $CO_2$ at $G$

So, $\angle AO_1K + \angle CO_2K = (\angle O_1GO_2 + \angle O_1O_2G) + (\angle GO_1O_2 + \angle O_1GO_2) = 180 + \angle O_1GO_2$

So, we have $\angle O_1TO_2 = 360 - 2 \angle ABC - (180 + \angle O_1GO_2) = 180 - 2 \angle ABC - \angle O_1GO_2$

Leave this for now and look at the other angle.

$\angle O_1PO_2 = \angle APC - \angle APO_1 - \angle O_2PC = 180 - \angle ABC - \angle PAU - (90 - \angle PVC) = 90 - \angle ABC - \angle PAU + \angle PVC$

So, showing $\angle O_1PO_2 = \angle O_1QO_2$ is equivalent to $90 - \angle ABC = \angle O_1GO_2 + \angle PVC - \angle PAU$

See that $\angle O_1GO_2 = \angle AUB - \angle VCB$

Let $W = PV \cap BC$ and $M = PA \cap BC$

We have $\angle PAU - \angle AUB = \angle PMC$ and $\angle PVC - \angle VCB = \angle PWC$

So, we just want to show $90 - \angle ABC = \angle PWC - \angle PMC = \angle MPW = \angle APV$

But $\angle APV = \angle APC - \angle VPC = 180 - \angle ABC - 90 = 90 - \angle ABC$, as desired.

So after all of that insane angle chase, $O_1O_2TQ$ is indeed cyclic, as claimed.

Now, we're basically done.

Let $O_3$ be circumcenter of $(BEP)$. Let perpendicular bisectors of $BE, CF$ meet at $I$ and those of $AD, BE$ meet at $J$.

By symmetry, we also have $(O_2O_3IQ)$ is also cyclic. So, we have that $Q$ must be the miquel point of the quadrilateral $O_3O_1JI$. So, $\omega = (IJT)$ also passes through $Q$.

So, $\omega_A, \omega_B, \omega_C, \omega$ all have the common point $Q$ and so we are done.
This post has been edited 3 times. Last edited by lneis1, Jul 21, 2021, 5:03 PM
Z K Y
The post below has been deleted. Click to close.
This post has been deleted. Click here to see post.
lvym
10 posts
#8 • 2 Y
Y by jhu08, centslordm
It takes time for me to understand that it's actually A-median line :roll:
Z K Y
The post below has been deleted. Click to close.
This post has been deleted. Click here to see post.
KST2003
173 posts
#9 • 3 Y
Y by Snark_Graphique, jhu08, centslordm
Let $\triangle XYZ$ be the orthic triangle of $\triangle ABC$. Let $A'$ be the point on $\overline{BC}$ such that $PA \perp PA'$. Define $B'$ and $C'$ respectively. Finally, let $O_A$, $O_B$ and $O_C$ be the centers of $\omega_A$, $\omega_B$ and $\omega_C$ respectively. It is well-known that $A'$, $B'$, $C'$ lie on a single line $\ell$ (This line is called the orthotransveral of $P$.) We will first show that $\omega_A$, $\omega_B$ and $\omega_C$ are concurrent at a point.

Since $X$ is the reflection of $A$ over the $A$-midline in $\triangle ABC$, $APDX$ is an isosceles trapezoid and hence $X$ lies on $\omega_A$. Now as $\angle AXA' = \angle APA' = 90^\circ$, it follows that $A'$ also lies on $\omega_A$. Thus $O_A$ is the midpoint of $AA'$. Similarly, $O_B$ and $O_C$ are midpoints of $BB'$ and $CC'$. This means that $O_A$, $O_B$ and $O_C$ all lie on the Gauss line $k$ of quadrilateral $\mathcal{P}(\overline{AB}, \overline{BC}, \overline{CA}, \ell)$, and hence $\omega_A$, $\omega_B$ and $\omega_C$ are concurrent at the point $P'$ which is the reflection of $P$ over $k$.

Let the perpendicular bisectors of $AD$, $BE$ and $CF$ be $\ell_A$, $\ell_B$ and $\ell_C$ respectively. We now claim that $P'$ is the Miquel point of quadrilateral $\mathcal{P}(\ell_A, \ell_B, \ell_C, k)$. To see this, let $U = \ell_A \cap \ell_C$. We just need to show that $O_A$, $O_C$, $U$, $P'$ are concyclic. This follows from angle chasing like so:
\begin{align*}
    \measuredangle O_AP'O_C &= \measuredangle O_CPO_A\\
    &= \measuredangle O_CPC + \measuredangle CPA + \measuredangle APO_A\\
    &= 90^\circ - \measuredangle CZP + \measuredangle AHC + \measuredangle AXP - 90^\circ \\
    &= \measuredangle AHC + \measuredangle DAX - \measuredangle FCZ\\
    &= \measuredangle AHC + \measuredangle CAH + \measuredangle DAC - \measuredangle ACH - \measuredangle FCA\\
    &= \measuredangle (\overline{AD}, \overline{CF}) = \measuredangle O_AUO_C.
\end{align*}Therefore $P'$ lies on the circumcircle of the triangle formed by $\ell_A$, $\ell_B$ and $\ell_C$.
Z K Y
The post below has been deleted. Click to close.
This post has been deleted. Click here to see post.
MP8148
888 posts
#10 • 5 Y
Y by amar_04, jhu08, lneis1, centslordm, suh
Similar to above solutions, but whatever.

[asy]
size(12cm);
defaultpen(fontsize(10pt));

pair A = dir(100), B = dir(220), C = dir(320), P = dir(60), M = (B+C)/2, N = (C+A)/2, K = (A+B)/2, X = foot(A,B,C), Y = foot(B,C,A), Z = foot(C,A,B), H = orthocenter(A,B,C), Oa = circumcenter(A,P,X), Ob = circumcenter(B,P,Y), Oc = circumcenter(C,P,Z), D = 2*foot(P,N,K)-P, E = 2*foot(P,K,M)-P, F = 2*foot(P,N,M)-P, P1 = intersectionpoint(H--H+dir(P--H)*100, circumcircle(X,Y,Z)), R = extension((B+E)/2,Ob,(C+F)/2,Oc), S = extension((C+F)/2,Oc,(A+D)/2,Oa), T = extension((A+D)/2,Oa,(B+E)/2,Ob);

draw(A--B--C--A, heavyblue);
draw(unitcircle, heavyblue);
draw(circumcircle(X,Y,Z), heavyblue);
draw(circumcircle(A,X,P)^^circumcircle(B,Y,P)^^circumcircle(C,Z,P), purple);
draw(P--P1, red);
draw(A--X^^B--Y^^C--Z, heavyblue+dotted);
draw(R--S--T--R, magenta);
draw(circumcircle(R,S,T), magenta+dashed);
draw(Oc--S--Oa, magenta);
draw(R--Ob, magenta);
draw(Ob--Oc, magenta);
draw(circumcircle(P1,Ob,Oc), heavycyan);

dot("$A$", A, dir(100));
dot("$B$", B, dir(240));
dot("$C$", C, dir(315));
dot("$M$", M, dir(270));
dot("$N$", N, dir(45));
dot("$K$", K, dir(135));
dot("$X$", X, dir(270));
dot("$Y$", Y, dir(30));
dot("$Z$", Z, dir(180));
dot("$P$", P, dir(45));
dot("$P'$", P1, dir(270));
dot("$H$", H, dir(120));
dot("$D$", D, dir(315));
dot("$E$", E, dir(225));
dot("$F$", F, dir(45));
dot("$R$", R, dir(180));
dot("$S$", S, dir(90));
dot("$T$", T, dir(180));
dot("$O_A$", Oa, dir(60));
dot("$O_B$", Ob, dir(135));
dot("$O_C$", Oc, dir(270));
[/asy]

Let $H$, $MNK$, and $XYZ$ be the orthocenter, medial triangle, and orthic triangle of $ABC$, respectively. Let $O_A$ denote the center of $\omega_A$; similar for $O_B$ and $O_C$. Let the perpendicular bisectors of $\overline{BE}$ and $\overline{CF}$ meet at $R$; similar for $S$ and $T$.

To show that $\omega_A$, $\omega_B$, and $\omega_C$ are coaxial, note that by symmetry $APDX$ is an isosceles trapezoid, so $X$ lies on $\omega_A$. Similarly, $Y$ lies on $\omega_B$ and $Z$ lies on $\omega_C$. Since $AH \cdot HX = BH \cdot HY = CH \cdot HZ$, we know that $H$ has equal power wrt $\omega_A$, $\omega_B$, and $\omega_C$, so the circles must be coaxial. The second intersection point of these circles, $P'$, is the intersection point of line $\overline{PH}$ and $(XYZ)$ not on segment $\overline{PH}$.

Now we prove that $P'$, $R$, $S$, $T$ are concyclic. I claim that $P'RO_BO_C$ is cyclic, so that by symmetry $P'SO_CO_A$ and $P'TO_AO_B$ are both cyclic, and we are done by Miquel points. Indeed \begin{align*} \measuredangle O_BRO_C &= \measuredangle(\overline{BE}, \overline{CF}) \\ &= (180^\circ - 2\measuredangle KMN) - \measuredangle YPZ \\ &= (180^\circ - 2\measuredangle CAB) - \measuredangle YPZ \\ &= (90^\circ - \measuredangle YBP) - (90^\circ - \measuredangle ZCP) - \measuredangle YPZ \\ &= (\measuredangle YPO_B - \measuredangle ZPO_C) - \measuredangle YPZ \\ &= \measuredangle O_CPO_B \\ &= \measuredangle O_BP'O_C. \end{align*}$\blacksquare$
Z K Y
The post below has been deleted. Click to close.
This post has been deleted. Click here to see post.
v_Enhance
6870 posts
#11 • 9 Y
Y by jhu08, 606234, lneis1, HamstPan38825, v4913, centslordm, Mathematicsislovely, TheHU-1729, Funcshun840
Call the formed triangle $XYZ$. Also denote by $O_a$, $O_b$, $O_c$ the centers of $PAD$, $PBE$, $PCF$. Let $L_a$, $L_b$, $L_c$ be the altitude feet. Also, let ray $PH$ meet the nine-point circle again at $Q$. We contend that $Q$ is the desired point.
[asy] import graph; size(12cm); pen zzttqq = rgb(0.6,0.2,0); pen qqwuqq = rgb(0,0.39215,0); pen zzttff = rgb(0.6,0.2,1); pen ccqqqq = rgb(0.8,0,0); pen fuqqzz = rgb(0.95686,0,0.6); pen cqcqcq = rgb(0.75294,0.75294,0.75294); draw((-1.23265,1.71708)--(-2.19748,-0.20728)--(0.20251,-0.20728)--cycle, linewidth(1) + zzttqq);
draw((-1.23265,1.71708)--(-2.19748,-0.20728), linewidth(1) + zzttqq); draw((-2.19748,-0.20728)--(0.20251,-0.20728), linewidth(1) + zzttqq); draw((0.20251,-0.20728)--(-1.23265,1.71708), linewidth(1) + zzttqq); draw(circle((-0.99748,0.39512), 1.34271), linewidth(1)); draw((-0.43383,-0.82356)--(-0.43383,2.33336), linewidth(1) + qqwuqq); draw((-0.43383,-0.82356)--(-0.56753,-0.92327), linewidth(1) + qqwuqq); draw((-0.43383,-0.82356)--(-1.82851,-0.12430), linewidth(1) + qqwuqq); draw((-1.23265,1.71708)--(-0.43383,2.33336), linewidth(1) + zzttqq); draw((-2.19748,-0.20728)--(-0.56753,-0.92327), linewidth(1) + zzttqq); draw(circle((0.14678,0.75490), 1.68186), linewidth(1) + linetype("2 2") + zzttff); draw(circle((-1.14082,-0.01508), 1.07399), linewidth(1) + linetype("2 2") + zzttff); draw(circle((-1.11506,0.45369), 0.67135), linewidth(1) + ccqqqq); draw((0.14678,0.75490)--(-1.14082,-0.01508), linewidth(1) + fuqqzz); draw((-0.43383,-0.82356)--(-1.51851,0.99031), linewidth(1)); draw((-0.45814,1.53901)--(-1.14082,-0.01508), linewidth(1) + fuqqzz); draw((-0.72900,1.89010)--(-0.79847,0.18963), linewidth(1) + fuqqzz); draw((-0.72900,1.89010)--(0.14678,0.75490), linewidth(1) + fuqqzz); dot("$A$", (-1.23265,1.71708), dir((1.108, 2.769))); dot("$B$", (-2.19748,-0.20728), dir((1.224, 2.749))); dot("$C$", (0.20251,-0.20728), dir((1.033, 2.749))); dot("$P$", (-0.43383,-0.82356), dir((0.978, 2.902))); dot("$D$", (-0.43383,2.33336), dir((0.978, 2.894))); dot("$E$", (-0.56753,-0.92327), dir((1.056, 2.904))); dot("$F$", (-1.82851,-0.12430), dir((1.157, 2.759))); dot("$Z$", (-0.45814,1.53901), dir((1.193, 2.300))); dot("$Y$", (-0.72900,1.89010), dir((1.141, 2.082))); dot("$X$", (-0.77259,0.82317), dir((1.070, 2.162))); dot("$H$", (-1.23265,0.51227), dir((1.108, 2.238))); dot("$Q$", (-1.51851,0.99031), dir((1.171, 2.894))); dot("$L_a$", (-1.23265,-0.20728), dir((1.108, 2.195))); dot("$P_a$", (-0.43383,0.40899), dir((0.978, 2.320))); dot("$O_a$", (0.14678,0.75490), dir((1.068, 2.343))); dot("$O_b$", (-1.14082,-0.01508), dir((1.063, 2.082))); dot("$O_c$", (-0.79847,0.18963), dir((1.166, 2.102))); dot("$L_b$", (-0.65526,0.94288), dir((1.244, 2.099))); [/asy]

Claim: $AQL_aPD$ is cyclic, and so on.
Proof. $ADPL_a$ is cyclic because it is an isosceles trapezoid, while $AQL_aP$ is cyclic by power of a point from $H$. $\blacksquare$

Claim: $Q$ is the Miquel point of complete quadrilateral $XYZO_aO_bO_c$.
Proof. It is sufficient to show that $Q$ lies on $(O_a O_b Z)$. This is an angle chase: \begin{align*} 		\measuredangle O_b Q O_a &= \measuredangle O_b Q P + \measuredangle P Q O_a 			= (90^{\circ} - \measuredangle P L_b Q) + (90 ^{\circ} - \measuredangle Q L_a P) \\ 		&= \measuredangle Q L_b P + \measuredangle P L_a Q 			= \measuredangle L_a P L_b + \measuredangle L_b Q L_a \\ 		&= \measuredangle L_a P L_b + 2 \measuredangle ACB \\ 	 		\measuredangle O_b Z O_a &=  \measuredangle(BE, AD) = \arg (AD) - \arg (BE) \\ 		&= (2\arg(BC) - \arg(PL_a)) - (2\arg(AC) - \arg(PL_b)) \\ 		&= 2\measuredangle ACB - \measuredangle L_b P L_a 	\end{align*}as needed. $\blacksquare$
Z K Y
The post below has been deleted. Click to close.
This post has been deleted. Click here to see post.
rafaello
1079 posts
#12
Y by
Relabel the problem introducing new points.
Quote:
Let $P$ be a point on the circumcircle of acute triangle $ABC$. Let $P_A,P_B,P_C$ be the reflections of $P$ in the $A$-midline, $B$-midline, and $C$-midline. Let $M_A$ be the midpoint of $BC$, $M_B$ the midpoint of $AC$ and $M_C$ the midpoint of $AB$. Let $H_A$ be the foot from $A$ to $BC$, $H_B$ be the foot from $B$ to $AC$, $H_C$ be the foot from $C$ to $AB$.

Let $O_A$ be the center of $(PP_AA)$, $O_B$ be the center of $(PP_BB)$, $O_C$ be the center of $(PP_CC)$.

Let $X$ be the intersection of perpendicular bisector of $P_AA$ and $P_BB$, $Y$ the intersection of perpendicular bisector of $P_BB$ and $P_CC$ and $Z$ the intersection of perpendicular bisector of $P_CC$ and $P_AA$.

Show that $(PP_AA), (PP_BB), (PP_CC),$ and $(XYZ)$ share a common point.

[asy]import olympiad;import geometry;
size(13cm);defaultpen(fontsize(10pt));pen org=magenta;pen med=mediummagenta;pen light=pink;pen deep=deepmagenta;

pair O,A,B,C,ma,mb,mc,ha,hb,hc,H,P,D,pa,pb,pc,X,Y,Z,oa,ob,oc; 
O=(0,0);A=dir(105);B=dir(200);C=dir(340);path w=circumcircle(A,B,C);ma=midpoint(B--C);mb=midpoint(A--C);mc=midpoint(A--B);
ha=foot(A,B,C);hb=foot(B,A,C);hc=foot(C,A,B);H=orthocenter(A,B,C);P=dir(90);path x=circumcircle(ma,mb,mc);D=intersectionpoints(x,100H-99P--P)[1];
path c=circumcircle(C,P,hc); path a=circumcircle(A,P,ha);path b=circumcircle(B,P,hb);
pc=2foot(P,ma,mb)-P;pa=2foot(P,mb,mc)-P;pb=2foot(P,ma,mc)-P;line ap=perpendicular(midpoint(A--pa),line(pa,A));line bp=perpendicular(midpoint(B--pb),line(pb,B));line cp=perpendicular(midpoint(C--pc),line(pc,C));X=intersectionpoint(ap,bp);Y=intersectionpoint(bp,cp);Z=intersectionpoint(ap,cp);
oa=circumcenter(A,P,ha);ob=circumcenter(B,P,hb);oc=circumcenter(C,P,hc);


draw(A--B--C--cycle,deep);draw(w,deep);draw(b,red);draw(a,red);draw(c,red);draw(x,deep);draw(circumcircle(X,Y,Z),lightblue+dashed);
draw(P--D,med);draw(C--hc,med);draw(B--hb,med);draw(A--ha,med);draw(ma--mb--mc--cycle,deep);
draw(Y--midpoint(C--pc),deep);draw(Y--midpoint(B--pb),deep);draw(X--midpoint(A--pa),deep);
draw(C--pc,deep);draw(B--pb,deep);draw(A--pa,deep);
draw(ob--Y--Z--oa--cycle,deepblue);draw(Y--X--Z--oc--oa,blue);


dot("$A$",A,dir(A)); 
dot("$B$",B,dir(B)); 
dot("$C$",C,dir(C));
dot("$M_A$",ma,dir(90));
dot("$M_B$",mb,dir(mb));
dot("$M_C$",mc,dir(mc));
dot("$H_A$",ha,dir(ha));
dot("$H_B$",hb,dir(hb));
dot("$H_C$",hc,dir(hc));
dot("$H$",H,dir(H));
dot("$P$",P,dir(P));
dot("$D$",D,dir(D));
dot("$P_A$",pa,dir(pa));
dot("$P_B$",pb,dir(pb));
dot("$P_C$",pc,dir(pc));
dot("$X$",X,dir(X));
dot("$Y$",Y,dir(Y));
dot("$Z$",Z,dir(Z));
dot("$O_A$",oa,dir(oa));
dot("$O_B$",ob,dir(ob));
dot("$O_C$",oc,dir(oc));
[/asy]


Let $D$ be intersection of $PH$ and $(M_AM_BM_C)$.

By reflections, $H_A$ lies on $(PP_AA)$, $H_B$ lies on $(PP_BB)$, $H_C$ lies on $(PP_CC)$.
Claim. $D$ lies on $(PP_AAH_A),(PP_BBH_B),(PP_CCH_C)$.
Proof.
By inversion at $H$ sending $(M_AM_BM_C)$ to $(ABC)$, this takes $D$ to $P$. By PoP, we get desired, the claim follows. $\square$

Claim. $O_AO_CDZ$ is cyclic.
Proof. We do angle chasing. We want $\measuredangle O_CDO_A=\measuredangle(YZ,XZ)$. Note that
\begin{align*}
\measuredangle(YZ,XZ)&=\measuredangle(CP_C,AP_A)\\&=\measuredangle P_CCB-\measuredangle(AP_A,BC)\\&=\measuredangle P_CCB-\measuredangle CH_AP\\&=\measuredangle P_CCB+\measuredangle PH_AA-90^\circ\\&=\measuredangle PH_AA+\measuredangle P_CCP-\measuredangle BAP-90^\circ
\end{align*}and
\begin{align*}
\measuredangle O_CDO_A&=\measuredangle CDP-\measuredangle CDO_C-\measuredangle O_ADP\\&=\measuredangle CDP+\measuredangle DP_CC+\measuredangle PP_AD-180^\circ\\&=\measuredangle CDP+\measuredangle DP_CC-\measuredangle DAP.
\end{align*}
Let $\ell$ be the parallel to $AB$ through $P$.
\begin{align*}
\measuredangle CDP+\measuredangle DP_CC+\measuredangle PP_AD-180^\circ&=\measuredangle P_CCB-\measuredangle CH_AP\Longleftrightarrow \\
\measuredangle DP_CC+\measuredangle PP_AD+\measuredangle CH_AP&=\measuredangle P_CCB+\measuredangle PP_CC\Longleftrightarrow \\
\measuredangle DP_CC+\measuredangle PP_AD+90^\circ-\measuredangle PH_AA&=\measuredangle P_CCB+\measuredangle PP_CC\Longleftrightarrow \\
\measuredangle DP_CC+90^\circ+\measuredangle AP_AD&=360^\circ-\measuredangle CBP-\measuredangle BPP_C\Longleftrightarrow \\
\measuredangle DP_CC+\measuredangle A_PAD&=270^\circ-\measuredangle CBP-\measuredangle BPC-\measuredangle CPP_C\Longleftrightarrow \\
\measuredangle DP_CC+\measuredangle CPP_C+\measuredangle A_PAD&=90^\circ+\measuredangle PCB\Longleftrightarrow \\
\measuredangle DPP_C+\measuredangle AP_AD&=90^\circ+\measuredangle PCB\Longleftrightarrow \\
\measuredangle AP_AC&=90^\circ+\measuredangle PCB\Longleftrightarrow \\
\measuredangle PCB&=\measuredangle (AP,\ell)=\measuredangle APP_C-90^\circ.\\
\end{align*}The claim follows. $\square$

In the same manner, we get that $XO_BO_CD$ and $YO_BO_CD$ are cyclic. Note that $O_A,O_B,O_C$ are collinear as they all lie on the perpendicular bisector of $PD$. Thus, $D$ is the Miquel point of $O_BYZO_A$, hence $D$ lies on $(XYZ)$. $\blacksquare$
Z K Y
The post below has been deleted. Click to close.
This post has been deleted. Click here to see post.
JAnatolGT_00
559 posts
#13 • 1 Y
Y by PRMOisTheHardestExam
Denote by $\ell_A,\ell_B,\ell_C$ perpendicular bisectors of $AD,BE,CF.$ Note for example that projection of $A$ onto $BC$ lies on $\odot (ADP),$ so orthocenter of $\triangle ABC$ has equal powers wrt all $\odot (ADP),\odot (BEP),\odot (CFP),$ hence these circles concur at $P,Q.$ Denote by $O_A,O_B,O_C$ centers of respective circles. By $$(\ell_A;\ell_B)=(AD;BE)=\measuredangle DAB+\measuredangle ABE=90^{\circ}+\measuredangle CBA+\measuredangle ADP+90^{\circ}+\measuredangle BAC+\measuredangle PEB=$$$$=\measuredangle BPA+\frac{\measuredangle AO_AP}{2}+\frac{\measuredangle PO_BB}{2}=\measuredangle BPA+\measuredangle APO_A+\measuredangle O_BPB=\measuredangle O_BPO_A=\measuredangle O_AQO_B$$and analogous equalities $Q$ is the Miquel point of complete quadrilateral on $\overline{O_AO_BO_C},$ $\ell_A,$ $\ell_B,$ $\ell_C.$ The conclusion follows.
This post has been edited 1 time. Last edited by JAnatolGT_00, Oct 2, 2021, 4:51 PM
Z K Y
The post below has been deleted. Click to close.
This post has been deleted. Click here to see post.
Nuterrow
254 posts
#14 • 15 Y
Y by MrOreoJuice, Pranav1056, JAnatolGT_00, REYNA_MAIN, Executioner230607, SatisfiedMagma, 636510, e61442289, SPHS1234, 554183, GeoKing, CyclicISLscelesTrapezoid, a_n, PRMOisTheHardestExam, Funcshun840
Solved with: MrOreoJuice, Geometry06, Fakesolver19, KILLER1234, Pranav1056, SatisfiedMagma, Arifa Alam, trigocalc, youngmind_007, Executioner230607, Bahnhofstrasse($\text{he rickrolled us when we fell asleep}$) and L567($\text{who helped us in almost every single moment}$).

[asy]
  /* Geogebra to Asymptote conversion, documentation at artofproblemsolving.com/Wiki go to User:Azjps/geogebra */
import graph; size(13cm); 
real labelscalefactor = 0.5; /* changes label-to-point distance */
pen dps = linewidth(0.7) + fontsize(10); defaultpen(dps); /* default pen style */ 
pen dotstyle = black; /* point style */ 
real xmin = -8.3216779501558, xmax = 7.029833943019902, ymin = -5.060620219381386, ymax = 3.9807610183171507;  /* image dimensions */
pen ccqqqq = rgb(0.8,0,0); pen ccwwff = rgb(0.8,0.4,1); pen yqqqyq = rgb(0.5019607843137255,0,0.5019607843137255); pen ffttww = rgb(1,0.2,0.4); pen zzttff = rgb(0.6,0.2,1); pen fuqqzz = rgb(0.9568627450980393,0,0.6); pen wwqqcc = rgb(0.4,0,0.8); pen qqzzff = rgb(0,0.6,1); 
 /* draw figures */
draw((-3.545063002016111,2.109548845476104)--(-4.86,-0.94), linewidth(0.8) + ccqqqq); 
draw((-3.545063002016111,2.109548845476104)--(1.16,-0.9), linewidth(0.8) + ccqqqq); 
draw((1.16,-0.9)--(-4.86,-0.94), linewidth(0.8) + ccqqqq); 
draw(circle((-1.85326773298179,-0.42820618624068185), 3.049978988998248), linewidth(0.4)); 
draw((-3.545063002016111,2.109548845476104)--(0.624765569183903,-0.13003492554205065), linewidth(0.8) + ccwwff); 
draw((0.624765569183903,-0.13003492554205065)--(0.6148406331219773,1.3636679517777692), linewidth(0.8) + ccwwff); 
draw((0.6148406331219773,1.3636679517777692)--(-3.545063002016111,2.109548845476104), linewidth(0.8) + ccwwff); 
draw((-4.86,-0.94)--(-2.8896960082038667,-4.115248170516802), linewidth(0.8) + yqqqyq); 
draw((-2.8896960082038667,-4.115248170516802)--(0.6148406331219773,1.3636679517777692), linewidth(0.8) + yqqqyq); 
draw((0.6148406331219773,1.3636679517777692)--(-4.86,-0.94), linewidth(0.8) + yqqqyq); 
draw((1.16,-0.9)--(0.6148406331219773,1.3636679517777692), linewidth(0.8) + ffttww); 
draw((0.6148406331219773,1.3636679517777692)--(-1.8813439323869796,2.4399994372869385), linewidth(0.8) + ffttww); 
draw((-1.8813439323869796,2.4399994372869385)--(1.16,-0.9), linewidth(0.8) + ffttww); 
draw(circle((-1.6686189842489785,0.6016110507563848), 2.407263704823015), linewidth(0.8) + zzttff); 
draw(circle((-1.5713322194232993,-1.0982471981664503), 3.2924729227027547), linewidth(0.8) + blue); 
draw(circle((-1.6129919770106291,-0.3703405532097846), 2.8231230285163686), linewidth(0.8) + fuqqzz); 
draw((-3.545063002016111,2.109548845476104)--(-3.524859164954307,-0.9311286323252778), linewidth(0.8) + wwqqcc); 
draw((-4.86,-0.94)--(-3.093952423074317,1.8210002497496307), linewidth(0.8) + wwqqcc); 
draw((-3.9016610634050766,1.2825410054996822)--(1.16,-0.9), linewidth(0.8) + wwqqcc); 
draw(circle((-2.69589763451716,0.3488775158583927), 1.5249894944991238), linewidth(0.8) + red); 
draw(circle((-4.6380660192823795,-0.7937726789482511), 1.989066858636808), linewidth(0.8) + green); 
draw((-1.6686189842489785,0.6016110507563848)--(-1.5713322194232993,-1.0982471981664503), linewidth(0.8) + qqzzff); 
draw((-1.6686189842489785,0.6016110507563848)--(-3.086604905938963,-2.0385037269704767), linewidth(0.8) + qqzzff); 
draw((-4.211595325029818,-2.73658242647142)--(-1.5713322194232993,-1.0982471981664503), linewidth(0.8) + qqzzff); 
draw((-1.6129919770106291,-0.3703405532097846)--(-4.211595325029818,-2.73658242647142), linewidth(0.8) + qqzzff); 
 /* dots and labels */
dot((-3.545063002016111,2.109548845476104),linewidth(2pt) + dotstyle); 
label("$A$", (-3.487050482775313,2.175624139262755), NE * labelscalefactor); 
dot((-4.86,-0.94),linewidth(2pt) + dotstyle); 
label("$B$", (-5.198006307270355,-1.1991969824476365), NE * labelscalefactor); 
dot((1.16,-0.9),linewidth(2pt) + dotstyle); 
label("$C$", (1.2220022451926937,-0.8381696066367573), NE * labelscalefactor); 
dot((0.6148406331219773,1.3636679517777692),linewidth(2pt) + dotstyle); 
label("$P$", (0.672612760263093,1.422175702787877), NE * labelscalefactor); 
dot((-1.85,-0.92),linewidth(2pt) + dotstyle); 
dot((-1.1925315010080557,0.604774422738052),linewidth(2pt) + dotstyle); 
dot((-4.202531501008056,0.584774422738052),linewidth(2pt) + dotstyle); 
dot((0.624765569183903,-0.13003492554205065),linewidth(2pt) + dotstyle); 
label("$D$", (0.6883096026896531,-0.06902432773531933), NE * labelscalefactor); 
dot((-2.8896960082038667,-4.115248170516802),linewidth(2pt) + dotstyle); 
label("$E$", (-2.827783100859792,-4.056022304081549), NE * labelscalefactor); 
dot((-1.8813439323869796,2.4399994372869385),linewidth(2pt) + dotstyle); 
label("$F$", (-1.8231851855599506,2.505257830220514), NE * labelscalefactor); 
dot((-3.524859164954307,-0.9311286323252778),linewidth(2pt) + dotstyle); 
label("$X$", (-3.455656797922193,-0.8695632914898773), NE * labelscalefactor); 
dot((-3.093952423074317,1.8210002497496307),linewidth(2pt) + dotstyle); 
label("$Y$", (-3.0318420524050724,1.8773841331581158), NE * labelscalefactor); 
dot((-3.538527536052532,1.1259612179574683),linewidth(2pt) + dotstyle); 
label("$H$", (-3.471353640348753,1.1867230663894774), NE * labelscalefactor); 
dot((-3.9016610634050766,1.2825410054996822),linewidth(2pt) + dotstyle); 
label("$Z$", (-4.083530494984593,1.4378725452144367), NE * labelscalefactor); 
dot((-4.024112063705148,1.098170107860692),linewidth(2pt) + dotstyle); 
label("$T$", (-4.366073658662674,0.966967272417638), NE * labelscalefactor); 
dot((-2.7435801672858964,-1.3998339998659126),linewidth(2pt) + dotstyle); 
label("$P_B$", (-2.6865115190207516,-1.340468564286676), NE * labelscalefactor); 
dot((-4.211595325029818,-2.73658242647142),linewidth(2pt) + dotstyle); 
label("$P_A$", (-4.146317864690833,-2.6747001705442726), NE * labelscalefactor); 
dot((-3.086604905938963,-2.0385037269704767),linewidth(2pt) + dotstyle); 
label("$P_C$", (-3.016145209978512,-1.9683422613490744), NE * labelscalefactor); 
dot((-1.6686189842489785,0.6016110507563848),linewidth(2pt) + dotstyle); 
label("$O_1$", (-1.6034293915881102,0.6687272663129988), NE * labelscalefactor); 
dot((-1.6129919770106291,-0.3703405532097846),linewidth(2pt) + dotstyle); 
label("$O_3$", (-1.5563388643084302,-0.3044769641337187), NE * labelscalefactor); 
dot((-1.5713322194232993,-1.0982471981664503),linewidth(2pt) + dotstyle); 
label("$O_2$", (-1.5092483370287502,-1.0422285581820367), NE * labelscalefactor); 
clip((xmin,ymin)--(xmin,ymax)--(xmax,ymax)--(xmax,ymin)--cycle); 
 /* end of picture */
[/asy]


Let $X$ be the feet of the perpendicular from $A$ to $BC$. In light of definition of $X$, define $Y$ and $Z$ for points $B$ and $C$ respectively. Also, let $\Gamma$ denote the Nine Point Circle of $\triangle ABC$. Denote midpoints of $HA$, $HB$ and $HC$ by $H_a$, $H_b$ and $H_c$ respectively.

Claim: $X$ lies on $\odot(APD)$. Similar claims follow for points $Y$ and $Z$.

Proof: This is all due to symmetry. Notice that $PD \parallel AX$ as perpendicular bisector $PD$ is also the perpendicular bisector of $AX$. Combining all this, we can also see that $APDX$ is an isosceles trapezium. $\square$

With some motivation of feet of perpendiculars, we introduce the orthocenter of $\triangle ABC$ as $H$. Define $T=\odot(APD) \cap \odot(CFP) \ne P$.

Claim: $H$ has equal powers with respect to all three circumcircles of $\triangle ADP, \triangle BEP$ and $\triangle CFP$.

Proof: We use Power of a Point for this one. Note that
\begin{align*}
    \text{Pow}_{\odot(APD)}(H)&=AH \cdot HX \\
    \text{Pow}_{\odot(CPF)}(H)&=CH \cdot HY \\
    \text{Pow}_{\odot(BEP)}(H)&=BH \cdot HZ
\end{align*}Now all these expressions are equal because $AZXC$, $BCZY$ and $ABXY$ are all cyclic. So, $H$ lies on the radical of all three circles. $\square$

Claim: $T$ lies on $\odot(BEP)$ as well as on $\Gamma$.

Proof: We use Power of a Point again. Observe that
\[TH \cdot HP = AH \cdot HX= BH \cdot HY\]which is sufficient to conclude that $T$ lies on all three circles. For Nine Point Circle, let $W=\Gamma \cap PH \ne T$. By a homothety centered at $H$ such $\Gamma \mapsto \odot (ABC)$ with a scale factor of 2, we get that $W$ is the midpoint of $PH$. Now, again we have
\[AH \cdot HX=TH \cdot HP \implies  \frac{AH}{2} \cdot HX=\frac{HP}{2}\cdot TH=H_aH \cdot HX=TH \cdot HW\]As $H_a$, $W$ and $Y$ lies on $\Gamma$, so $T$ also lies on it by Power of a Point. $\square$

Define $O_1$, $O_2$ and $O_3$ as the centers of the circumcircles of $\triangle ADP, \triangle BEP$ and $\triangle CFP$. Now, as these circles as coaxial with two common points, then $O_1-O_2-O_3$ are collinear.

Until now, we have shown that three circumcircles of $\triangle ADP, \triangle BEP$ and $\triangle CFP$ concur at a point $T$. We now wish to show that $\omega$ passes through $T$. Define $\ell_a$ as the perpendicular bisector of $AD$. Similarly, define $\ell_b$ and $\ell_c$. From here on, we will show two approaches to finish it off.

Solution with Miquel Points Define $P_B$ as the intersection of perpendicular bisectors of $AD$ and $CF$ and similarly define $P_A$ and $P_C$. We claim the following:

Claim: Points $T$, $P_C$, $O_1$ and $O_2$ are concylic and similarly points $T$, $P_B$, $O_3$ and $O_1$ and $T$, $P_A$, $O_2$ and $O_3$ are concylic.

Proof: Here is a long angle chase. First let $K= AD \cap BE$, \begin{align*} \measuredangle O_1PO_2 &= \measuredangle APO_2 - \measuredangle APO_1 \\&= \measuredangle APB + \measuredangle BPO_2 - \measuredangle APO_1 \\&= \measuredangle ACB + 90^{\circ} - \measuredangle PEB - (90^{\circ} - \measuredangle PDA) \\&=  \measuredangle ACB - \measuredangle PDA - \measuredangle PEB \\&= \measuredangle ACB + \measuredangle DAH + \measuredangle EBH \\&= \measuredangle YHX + \measuredangle DAH + \measuredangle EBH \\&= \measuredangle AHB + \measuredangle KAH + \measuredangle KBH \\&= \measuredangle(AD, BE) \\&= \measuredangle O_2P_CO_1 \end{align*}Therefore $T$ must be the miquel point of complete quadrilateral $P_AP_CO_2O_3O_1P_B$ and points $T$, $P_A$, $P_B$ and $P_C$ are concyclic, and we are done. $\blacksquare$

An approach with Simson's line by SatisfiedMagma Now, project from $T$ to $\ell_a$ and call that point $X_a$. Similarly, define $X_b$ and $X_c$. By Simson Line Theorem, it is sufficient to show that $X_a-X_b-X_c$ are collinear. This can be rephrased as proving that the reflections of $T$ along the perpendicular bisectors are collinear. Call the reflection of $T$ across $\ell_a$ as $Y_a$. Analogous definitions go for $Y_b$ and $Y_c$. We would show that $P$ also lies on the line $Y_aY_bY_c$. We use directed angles for this one to avoid configuration issues that are all over the place.

[asy]
 /* Geogebra to Asymptote conversion, documentation at artofproblemsolving.com/Wiki go to User:Azjps/geogebra */
import graph; size(27.389825480105902cm); 
real labelscalefactor = 0.5; /* changes label-to-point distance */
pen dps = linewidth(0.7) + fontsize(10); defaultpen(dps); /* default pen style */ 
pen dotstyle = black; /* point style */ 
real xmin = -4.645642645074283, xmax = 22.74418283503162, ymin = -9.582906939384474, ymax = 4.635985070722677;  /* image dimensions */
pen zzttqq = rgb(0.6,0.2,0.); pen ffqqff = rgb(1.,0.,1.); pen ffxfqq = rgb(1.,0.4980392156862745,0.); pen qqwuqq = rgb(0.,0.39215686274509803,0.); 

draw((5.616481742324597,3.927473792624218)--(2.28,-3.58)--(12.62,-3.56)--cycle, linewidth(0.4) + zzttqq); 
draw((11.23067158888095,-3.0574463960736944)--(10.725432586193207,-3.05842364752957)--(10.726409837649083,-3.5636626502173128)--(11.231648840336826,-3.5626853987614373)--cycle, linewidth(0.4) + qqwuqq); 
 /* draw figures */
draw((5.616481742324597,3.927473792624218)--(2.28,-3.58), linewidth(0.4) + zzttqq); 
draw((2.28,-3.58)--(12.62,-3.56), linewidth(0.4) + zzttqq); 
draw((12.62,-3.56)--(5.616481742324597,3.927473792624218), linewidth(0.4) + zzttqq); 
draw((xmin, 2.2501168513494103*xmin-20.33337054255311)--(xmax, 2.2501168513494103*xmax-20.33337054255311), linewidth(1.2) + green); /* line */
draw(circle((7.445765256598296,-1.3806376613196467), 5.614474631083801), linewidth(0.4)); 
draw((xmin, 0.00193423597678917*xmin + 0.16610006677407735)--(xmax, 0.00193423597678917*xmax + 0.16610006677407735), linewidth(1.2) + green); /* line */
draw((xmin, -1.0691017738717867*xmin + 4.394808215344811)--(xmax, -1.0691017738717867*xmax + 4.394808215344811), linewidth(1.2) + green); /* line */
draw(circle((7.766578625433584,0.18112246256795334), 4.3194982225912915), linewidth(0.4) + blue); 
draw(circle((8.529454305614609,-1.1411016766748934), 4.752224024226958), linewidth(0.4) + blue); 
draw(circle((6.535358242863149,-0.9159442730280686), 2.8072373155419), linewidth(0.4) + red); 
draw((5.616481742324597,3.927473792624218)--(11.229401703645303,-2.400915729243627), linewidth(0.4)); 
draw((2.28,-3.58)--(0.8661125589615923,-6.9075500120045215), linewidth(0.4)); 
draw((12.62,-3.56)--(9.634681807937246,3.480814088951904), linewidth(0.4)); 
draw(circle((1.8106850010207078,-2.869890758757947), 2.403208652049753), linewidth(0.4) + ffqqff); 
draw(shift((13.924572144284497,-10.491976564515417))*xscale(13.541472610165293)*yscale(13.541472610165293)*arc((0,0),1,79.35432995589181,200.60523367494213), linewidth(0.4) + blue); 
draw((0.4245633726820106,-9.43308864858062)--(3.7914990318905453,-1.5090867315754948), linewidth(0.4)); 
draw((4.9709615871285795,-4.290825910464538)--(3.7914990318905453,-1.5090867315754948), linewidth(0.4)); 
draw((5.613827365770179,-3.56370422354009)--(3.7914990318905453,-1.5090867315754948), linewidth(0.4)); 
draw((0.21466245000390777,-4.666588797364495)--(8.688732552133265,-8.267250319702413), linewidth(0.4) + linetype("4 4") + ffxfqq); 
draw((0.21466245000390777,-4.666588797364495)--(13.524461437570075,0.9767905829586241), linewidth(0.4) + linetype("4 4") + ffxfqq); 
draw((1.625198830292137,-5.265930555120951)--(12.791987530928656,4.638373057206871), linewidth(0.4) + linetype("4 4") + ffxfqq); 
draw((3.7914990318905453,-1.5090867315754948)--(11.219387288079853,2.7765371180949248), linewidth(0.4)); 
draw((3.7914990318905453,-1.5090867315754948)--(0.19518118399729972,-3.584032531558999), linewidth(0.4)); 
draw((0.19518118399729972,-3.584032531558999)--(2.28,-3.58), linewidth(0.4)); 
draw((11.231648840336826,-3.5626853987614373)--(11.219387288079853,2.7765371180949248), linewidth(0.4)); 
draw((2.28,-3.58)--(11.219387288079853,2.7765371180949248), linewidth(0.4)); 
 /* dots and labels */
dot((5.616481742324597,3.927473792624218),dotstyle); 
label("$A$", (5.357597965051351,4.183457519312232), NE * labelscalefactor); 
dot((2.28,-3.58),dotstyle); 
label("$B$", (1.8802809910552971,-3.4142418964736665), NE * labelscalefactor); 
dot((12.62,-3.56),dotstyle); 
label("$C$", (12.860028422645577,-3.6285970524049302), NE * labelscalefactor); 
dot((7.45,-3.57),linewidth(4.pt) + dotstyle); 
label("$M_a$", (7.548784003459824,-3.3904246569257483), NE * labelscalefactor); 
dot((9.118240871162298,0.18373689631210888),linewidth(4.pt) + dotstyle); 
label("$M_b$", (9.477980406841196,0.3965164311932416), NE * labelscalefactor); 
dot((3.948240871162298,0.17373689631210887),linewidth(4.pt) + dotstyle); 
label("$M_c$", (3.738025675792915,0.8014095035078506), NE * labelscalefactor); 
dot((11.219387288079853,2.7765371180949248),dotstyle); 
label("$P$", (11.240456133387141,3.0402300210121593), NE * labelscalefactor); 
dot((9.634681807937246,3.480814088951904),dotstyle); 
label("$F$", (9.31125973000577,2.992595541916323), NE * labelscalefactor); 
dot((11.229401703645303,-2.400915729243627),dotstyle); 
label("$D$", (11.359542331126733,-2.7473591891319575), NE * labelscalefactor); 
dot((0.8661125589615923,-6.9075500120045215),dotstyle); 
label("$E$", (0.37979489953645207,-6.915376110017639), NE * labelscalefactor); 
dot((3.7914990318905453,-1.5090867315754948),linewidth(4.pt) + dotstyle); 
label("$T$", (3.880929113080424,-1.318324816256867), NE * labelscalefactor); 
dot((5.624951229128003,-0.45125088473648656),linewidth(4.pt) + dotstyle); 
label("$H$", (5.714856558270124,-0.2703662761484673), NE * labelscalefactor); 
dot((0.21466245000390777,-4.666588797364495),linewidth(4.pt) + dotstyle); 
dot((4.211730889888056,-2.9718243883411195),linewidth(4.pt) + dotstyle); 
dot((1.625198830292137,-5.265930555120951),linewidth(4.pt) + dotstyle); 
dot((4.381230309509562,-2.8999563210200168),linewidth(4.pt) + dotstyle); 
label("$X_c$", (3.6427567176012423,-2.914079865967385), NE * labelscalefactor); 
dot((4.702663198830361,-2.5363954775577926),linewidth(4.pt) + dotstyle); 
label("$X_{a}$", (4.6668980181617234,-2.1757454399819216), NE * labelscalefactor); 
dot((2.1080312022862815,-5.471087690078049),linewidth(4.pt) + dotstyle); 
label("$X_b$", (2.1899051051782337,-5.962686528100911), NE * labelscalefactor); 
dot((0.4245633726820106,-9.43308864858062),linewidth(4.pt) + dotstyle); 
label("$Y_b$", (-0.0012809332302387774,-9.178013867069865), NE * labelscalefactor); 
dot((5.613827365770179,-3.56370422354009),linewidth(4.pt) + dotstyle); 
label("$Y_a$", (5.714856558270124,-3.36660741737783), NE * labelscalefactor); 
dot((4.9709615871285795,-4.290825910464538),linewidth(4.pt) + dotstyle); 
label("$Y_c$", (4.6668980181617234,-4.724190071609167), NE * labelscalefactor); 
label("$l_b$", (4.285822185395033,-6.796289912278048), NE * labelscalefactor,ffxfqq); 
label("$l_c$", (7.000987493857706,-2.1757454399819216), NE * labelscalefactor,ffxfqq); 
label("$l_a$", (7.453515045268151,-0.5799903902714036), NE * labelscalefactor,ffxfqq); 
dot((0.19518118399729972,-3.584032531558999),linewidth(4.pt) + dotstyle); 
label("$O$", (0.2845259413447793,-3.3904246569257483), NE * labelscalefactor); 
dot((11.231648840336826,-3.5626853987614373),linewidth(4.pt) + dotstyle); 
clip((xmin,ymin)--(xmin,ymax)--(xmax,ymax)--(xmax,ymin)--cycle); 
 /* end of picture */
[/asy]

We start by noticing that $ADY_aT$ is an isosceles trapezium as $AD$ and $Y_a$ share the same perpendicular bisector. So $\measuredangle (PY_a,PA)=\measuredangle Y_aPA =\measuredangle DPT= 90^\circ+\measuredangle (PT,BC)$. Now, in a similar fashion, we have that $\measuredangle (PY_b,PB)=90^\circ+\measuredangle (PT,CA)$. As, $ABCP$ is cyclic, $\measuredangle (PA,PB)= \measuredangle (CA,BC)$. Now, we will move to the critical part. Notice the following for finish:
\begin{align*}
    \measuredangle (PY_a,PY_b)&= \measuredangle (PA,PB)+ \measuredangle (PB,Y_bB)+\measuredangle (PY_a,PA) \\
    &=\measuredangle (CA,CB)+ 90^\circ +\measuredangle (CA,PT)+ 90^\circ + \measuredangle (PT,BC) \qquad \text{(}\bigstar\text{)} \\
    &= 0^\circ
\end{align*}This all comes down to zero if you properly focus on the triangle formed by the angles in ($\bigstar$). As these points are now collinear, we are done by symmetry. $\blacksquare$
A Short Story
Attachments:
This post has been edited 2 times. Last edited by Nuterrow, Nov 6, 2021, 12:28 PM
Reason: added some more content
Z K Y
The post below has been deleted. Click to close.
This post has been deleted. Click here to see post.
Pranav1056
35 posts
#15
Y by
@above tbh I haven't still done the chase :joy: due to IGO :blobsweat: .
I'll do it in this week uff too much hardwork
Z K Y
The post below has been deleted. Click to close.
This post has been deleted. Click here to see post.
primesarespecial
364 posts
#16
Y by
WLOG,Assume $AB<AC$.Let $H$ ,$Z$ and $T$ be the feet from $A$,$B$ nd $C$ onto the opp sides.Let $U$ be the orthocentre.
Let $X$ and $Y$ be the midpoints of $AB$ and $AC$ respectively. Also let the perpendicular bisectors of $AD$ and $CF$ meet at $L$ ,
of $FC$ and $BE$ at $K$,and of $AD$ and $BE$ at $J$.Then let the centres of circles $(APD),(CFP) ,(BEP)$ be $M,N,O$ respectively.

Now,let $AP$ meet $HD$ at $G$.Triangles $GPD$ and $GAH$ are homothetic.The midpoints of $DP$,$AH$ and $G$ are thus collinear,which implies
$G$ lies on $XY$ and thus $GPD$ is isosceles .$PA=DH$ and thus $D$ lies on $(APD)$.Similar conditions follow for $T$ and $Z$.

Now,$UA.UH=CU.UT=BU.UZ$,it follows that $U$ is the radical center of the circles $(APD),(CFP) ,(BEP)$,but $P$ is also the radical center ,implying that these circles are coaxial.
Let the second intersection of these circles be $I$.Also,$O--M--N$

Now,$ITH+IHT=TCP+HAP=90-A+ACP+90-C+PAC=2B$
Now,$MIN=NIP-MIP=PIT+PHI-180=PTC+AHP-B$
$=PTC+AHP+AUC-180$.
Also,$PADH$ and $PFTC$ are isc. traps.
So,$PTC+AHP+AUC-180=AUC+FCT+DAH-180$
$=FCU-(CT,AD)=(FC,AD).$
And $MLN=ALN-ALM=ALF+90-LFC-90+LAD=(AD,CF)$
Thus,$MNLI$ is cyclic. Similalry, $MOIJ$ is cyclic.
Now,
$ILK=IMN=IJK$,thus $IJKL$ is cyclic,which finishes.

Remark:The hours that I spent with the problem were the most obnoxious and unsavoury I have ever spent.The words that uttered out of my mouth after the completion of the angle chase were behench...
This post has been edited 1 time. Last edited by primesarespecial, Nov 7, 2021, 3:11 AM
Z K Y
The post below has been deleted. Click to close.
This post has been deleted. Click here to see post.
rama1728
800 posts
#17
Y by
Eyed wrote:
Let $P$ be a point on the circumcircle of acute triangle $ABC$. Let $D,E,F$ be the reflections of $P$ in the $A$-midline, $B$-midline, and $C$-midline. Let $\omega$ be the circumcircle of the triangle formed by the perpendicular bisectors of $AD, BE, CF$.

Show that the circumcircles of $\triangle ADP, \triangle BEP, \triangle CFP,$ and $\omega$ share a common point.

Extraordinary problem!! Solved with MathsCrazy

No idea how many hours this took lol, but I think less than 4.5 hours.
First of all let \(H_a, H_b, H_c\) be the foot of the \(A, B, C\) altitudes of triangle \(ABC\). First of all, \(PH_aAD\) is an isosceles trapezoid because the \(A\) midline bisects \(AH_a\) and \(PD\), and hence \(PH_aAD\) is cyclic. Similarly, \(PH_bBE\) and \(PH_cCF\) are cyclic. Note that the power of \(H\) wrt these three circles is equal because \[AH\cdot HH_a=BH\cdot HH_b=CH\cdot HH_c\]and so the three circles are concurrent at a point, say \(L\).

We now do something more interesting. Let \(L_1,L_2,L_3\) be the reflections of \(L\) over the perpendicular bisectors of \(AD, BE, CF\) respectively. If we prove \(L_1,L_2,L_3\) collinear, we will be done because of Steiner. Infact we will prove something stronger, which is \(P\) belongs in this line.

Now if we prove \(P,L_1,L_3\) collinear, the other results are analogous. We shall prove that \(\angle APL_1+\angle CPL_3=\angle APC\), if we do so, we are done. Now, \(\angle APL_1=\angle ADL_1=\angle LPD\). Analogously, \(\angle CPL_3=\angle LPF\), so we want to prove \(\angle APC=\angle FPD\) or \(\angle APD=\angle CPF\), which is true because of symmetry, so we are done!

PS: Why is the title so strange? Where is fish here?
Attachments:
This post has been edited 2 times. Last edited by rama1728, Feb 9, 2022, 10:54 AM
Reason: .
Z K Y
The post below has been deleted. Click to close.
This post has been deleted. Click here to see post.
cadaeibf
700 posts
#18
Y by
Let $M_aM_bM_c$ and $H_aH_bH_c$ be the medial and orthic triangle of $ABC$, and $H$ the orthocentre of $ABC$. Furthermore, let $Q$ be the point by a negative inversion in $H$ with power $-AH\cdot HH_a$. In other words, $Q$ belongs to the nine point circle $(H_aH_bH_cM_aM_bM_c)$ and is on the opposite side of $P$ wrt to $H$ on line $HP$.
By symmetry wrt to the $A$-midline (from now on call the three midlines $m_a,m_b,m_c$), $H_a$ belongs to $(ADP)$, and so $QH\cdot HP=AH\cdot HH_a$, which implies that $Q$ also belongs to $(ADP)$, and similarly to $(BEP)$ and $(CFP)$. Therefore their circumcenters $O_a$ $O_b$ and $O_c$ lies on the same line $\ell$, which is the axis of $PQ$ (since all three circles pass through $P$ and $Q$). Furthermore call the three axes of $AD,BE,CF$ $\ell_a,\ell_b,\ell_c$ respectively and $\ell_b\cap \ell_c=X$ and cyclically for $Y,Z$. Finally, let the three axes of $H_aP,H_bP,H_cP$ be called $\ell_a',\ell_b',\ell_c'$.
Let us now establish two lemmas:
1) $$\measuredangle (\ell_a,\ell)=\measuredangle (\ell_a,m_a)+\measuredangle (m_a,\ell)=\measuredangle (m_a,\ell_a')+\measuredangle (m_a,\ell)=
(HH_a,H_aP)+\measuredangle (HH_a,PQ)=\measuredangle HH_aP+\measuredangle H_aHP$$where in the second to last equality we use the fact that $\measuredangle (\ell,m)=\measuredangle (\ell',m')$ if $\ell'\perp \ell$ and $m\perp m'$.
2) $\measuredangle (QO_a,\ell)=\measuredangle QAP=\measuredangle QH_aP$ where in the first equality we used the fact that $\measuredangle (QO_a,\ell)$ is half the central angle corresponding to arc $\overarc QP$ of $(AQPH_a)$

By the first lemma, we have
$$\measuredangle O_bXO_c=\measuredangle (\ell_b,\ell_c)=\measuredangle (\ell_b,\ell)-\measuredangle (\ell_c,\ell)=(\measuredangle HH_bP+\measuredangle H_bHP)-(\measuredangle HH_cP+\measuredangle H_cHP)=(\measuredangle (H_bH,HP)-\measuredangle (H_cH,HP))+(\measuredangle HH_bP+\measuredangle PH_cH)=\measuredangle H_bHH_c+2\pi-(\measuredangle H_bPH_c+H_cHH_b)=2H_bHH_c-\measuredangle H_bPH_c=2(\pi-\measuredangle A)-H_bPH_c=-2\measuredangle A-\measuredangle H_bPH_c$$Instead, by the second lemma,
$$\measuredangle O_bQO_c=\measuredangle QO_b,\ell)-\measuredangle (QO_c,\ell)=\measuredangle QH_bP-\measuredangle QH_cP=\measuredangle QH_bP+\measuredangle PH_cQ=2\pi-(\measuredangle H_bPH_c+\measuredangle H_cQH_b)=-\measuredangle H_bPH_c+\measuredangle H_bH_aH_c=\pi-2\measuredangle A-\measuredangle H_bPH_c=-2\measuredangle A-\measuredangle H_bPH_c$$Since these two expressions are the same, it follows that $O_bO_cXQ$ is cyclic, and similarly for the other two quadrilaterals.
Therefore, since $YZ\cap O_bO_c=O_a$ and $Q$ belongs to both $(O_aO_bZ)$ and $(O_aO_cY)$, it follows that $Q$ is the miquel point of $O_bO_cYZ$, which means that $Q\in (XYZ)$, or in other words $\omega$ also passes through $Q$, as wanted.
Z K Y
The post below has been deleted. Click to close.
This post has been deleted. Click here to see post.
Leo.Euler
577 posts
#19 • 1 Y
Y by centslordm
Solved with apotosaurus and sixoneeight. This took over 4 hours, but it was totally worth it!
Claim: The circumcircles of $\triangle ADP$, $\triangle BEP$, and $\triangle CFP$ are coaxial.
Proof. We will show that the orthocenter $H$ of $\triangle ABC$ has equal power with respect to the three circles, which suffices since $H \neq P$. Let $h_A$, $h_B$, $h_C$ be the foot of the altitudes from $A$, $B$, $C$ in $\triangle ABC$. Note that $h_A \in (ADP)$, and analogous concyclicities hold for $B$, $C$, as $Ph_AAD$ is an isosceles trapezoid. Thus the powers of $H$ with respect to $(ADP)$, $(BEP)$, and $(CFP)$ are \[ AH \cdot Hh_A; BH \cdot Hh_B; CH \cdot Hh_C \]respectively, and elementary calculations show that these quantities are equal, finishing. :yoda:

By the above claim, the three circumcircles are concurrent at some point $Q \neq P$. Let $\Gamma$ denote the nine point circle of $\triangle ABC$.

Claim: $Q$ lies on $\Gamma$.
Proof. Let $Q'$ be the intersection of $\overline{PH}$ and $\Gamma$ different from $Q$. Noting the homothety of scale factor 2 centered at $H$ that maps $\Gamma$ to $(ABC)$, it's easy to see that $Q'$ is the midpoint of $PH$. Thus the PoP equation $AH \cdot Hh_A = QH \cdot HP$ rewrites as $(AH/2)\cdot Hh_A = QH \cdot (HP/2)$, so $Q$ is concyclic with $Q'$, the midpoint of $AH$, and $h_A$, which means that $Q$ lies on $\Gamma$, as required. :yoda:

Let $O_A$, $O_B$, $O_C$ be the circumcenters of $\triangle ADP$, $\triangle BEP$, $\triangle CFP$ respectively. As the circumcircles are coaxial, $O_A$, $O_B$, and $O_C$ are collinear. Let $X$ be the intersection of the perpendicular bisectors of $\overline{BE}$ and $\overline{CF}$, and similarly define $Y$, $Z$.

Claim: $Q$, $X$, $O_B$, and $O_C$ are concylic.
Proof. We omit this proof since I am lazy. However, this is nothing other than an angle chase which has steps that are rather straightforward. :yoda:

Using cyclic variants of the above claim, $Q$ is the Miquel point of the complete quadrilateral induced by the three perpendicular bisectors given in the problem statement and $\overline{O_AO_BO_C}$, and this implies $Q \in \omega$, as desired. :starwars:
Z K Y
The post below has been deleted. Click to close.
This post has been deleted. Click here to see post.
GrantStar
815 posts
#20 • 1 Y
Y by OronSH
[asy]
/* Geogebra to Asymptote conversion, documentation at artofproblemsolving.com/Wiki go to User:Azjps/geogebra */ import graph; size(21.5cm); real labelscalefactor = 1; /* changes label-to-point distance */ pen dps = linewidth(0.4) + fontsize(10); defaultpen(dps); /* default pen style */ pen dotstyle = black; /* point style */ real xmin = 1.5, xmax = 23., ymin = -17.16082195484213, ymax = 6.403086819243258;  /* image dimensions */ pen zzttqq = rgb(0.6,0.2,0.); pen ffwwzz = rgb(1.,0.4,0.6); pen ffqqtt = rgb(1.,0.,0.2); pen zzwwff = rgb(0.6,0.4,1.); pen zzccqq = rgb(0.6,0.8,0.);
draw((8.,5.)--(4.,-7.)--(16.,-7.)--cycle, linewidth(1.) + zzttqq); /* draw figures */ draw((8.,5.)--(4.,-7.), linewidth(1.) + zzttqq); draw((4.,-7.)--(16.,-7.), linewidth(1.) + zzttqq); draw((16.,-7.)--(8.,5.), linewidth(1.) + zzttqq); draw(circle((10.,-2.3333333333333335), 7.6011695006609195), linewidth(1.)); draw(circle((11.25859729693003,-1.), 6.827770964491974), linewidth(1.) + ffwwzz); draw(circle((11.640433313973183,-9.46064997095977), 8.026893515230881), linewidth(1) + ffqqtt); draw(circle((11.34700753997859,-2.958977380064229), 6.162808016468519), linewidth(1.) + zzwwff); draw((xmin, 3.051341078506704*xmin-35.353820418487146)--(xmax, 3.051341078506704*xmax-35.353820418487146), linewidth(1.) + zzccqq); /* line */ draw((xmin, 0.050255999895993386*xmin-10.045651586376124)--(xmax, 0.050255999895993386*xmax-10.045651586376124), linewidth(1.) + zzccqq); /* line */ draw((xmin, 1.455274712652225*xmin-19.4719905172692)--(xmax, 1.455274712652225*xmax-19.4719905172692), linewidth(1.) + zzccqq); /* line */ draw(circle((7.423363104467918,-6.726919991206829), 3.0659345971159127), linewidth(1.) + zzccqq); draw(circle((9.,-3.3333333333333335), 3.8005847503304597), linewidth(1.)); draw((xmin, -22.157810141843235*xmin + 248.46586136884505)--(xmax, -22.157810141843235*xmax + 248.46586136884505), linewidth(1.)); /* line */ draw((8.,5.)--(8.,-7.), linewidth(1.)); draw((5.367817773118896,-4.452125858859596)--(17.436390166623063,-3.907461367366656), linewidth(1.)); draw((4.,-7.)--(12.307692307692294,-1.4615384615384392), linewidth(1.)); draw((5.2,-3.4)--(16.,-7.), linewidth(1.)); /* dots and labels */ dot((8.,5.),dotstyle); label("$A$", (8.065570049789427,5.174886064078719), NE * labelscalefactor); dot((4.,-7.),dotstyle); label("$B$", (4.065144732967769,-6.826389886386216), NE * labelscalefactor); dot((16.,-7.),dotstyle); label("$C$", (16.066420683432742,-6.826389886386216), NE * labelscalefactor); dot((17.436390166623063,-3.907461367366656),dotstyle); label("$P$", (17.505170139482637,-3.7383422734010865), NE * labelscalefactor); dot((17.436390166623063,1.9074613673666603),dotstyle); label("$D$", (17.505170139482637,2.0868384510935893), NE * labelscalefactor); dot((4.285198890406509,-12.674922218177695),dotstyle); label("$E$", (4.363422059222016,-12.49365908521688), NE * labelscalefactor); dot((5.906411046281557,-0.06413499391948552),dotstyle); label("$F$", (5.977628766009702,0.10417151775654603), NE * labelscalefactor); dot((6.709048673381286,-9.70848163694446),linewidth(4.pt) + dotstyle); label("$X$", (6.784732119403545,-9.563522997895763), NE * labelscalefactor); dot((8.433006119182366,-9.621842431727583),linewidth(4.pt) + dotstyle); label("$Z$", (8.504213176633908,-9.475794372526867), NE * labelscalefactor); dot((9.950607469079369,-4.991123091989649),linewidth(4.pt) + dotstyle); label("$Y$", (10.013145532978918,-4.8437229530491726), NE * labelscalefactor); dot((5.367817773118896,-4.452125858859596),linewidth(4.pt) + dotstyle); label("$J$", (5.433711288722547,-4.317351200835798), NE * labelscalefactor); dot((8.,-7.),linewidth(4.pt) + dotstyle); label("$A_0$", (8.065570049789427,-6.861481336533775), NE * labelscalefactor); dot((5.2,-3.4),linewidth(4.pt) + dotstyle); label("$C_0$", (5.275799763058534,-3.26460769640905), NE * labelscalefactor); dot((12.307692307692294,-1.4615384615384392),linewidth(4.pt) + dotstyle); label("$B_0$", (12.381818417939108,-1.3170322132195647), NE * labelscalefactor); dot((8.,-4.333333333333334),linewidth(4.pt) + dotstyle); label("$H$", (8.065570049789427,-4.194531125319345), NE * labelscalefactor); dot((10.487540294650579,-6.830713540893745),linewidth(4.pt) + dotstyle); label("$G$", (10.557063010266072,-6.6860240857959825), NE * labelscalefactor); dot((11.25859729693003,-1.),linewidth(4.pt) + dotstyle); label("$O_A$", (11.329074913512358,-0.8608433613013069), NE * labelscalefactor); dot((11.640433313973183,-9.460649970959771),linewidth(4.pt) + dotstyle); label("$O_B$", (11.7150808651355,-9.317882846862855), NE * labelscalefactor); dot((11.347007539978591,-2.9589773800642294),linewidth(4.pt) + dotstyle); label("$O_C$", (11.416803538881252,-2.825964569564571), NE * labelscalefactor); clip((xmin,ymin)--(xmin,ymax)--(xmax,ymax)--(xmax,ymin)--cycle); /* end of picture */  [/asy]

Let $J$ be the intersection of $PH$ past $H$ and the nine point circle of $(ABC)$. Also, let $XYZ$ be the triangle formed and $A_0,B_0,C_0$ be the feet from the altitudes.

Claim: $J$ lies on $(ADP)$, $(BEP)$, $CFP$.
Proof. By symmetry $A_0$ lies on $(ADP)$. Then, power of a point and scaling by $2$ implies the result. $\blacksquare$
Let $O_A,O_B,O_C$ be the circumcenters of the three triangles.

Claim: $J$ is the miquel point of the quadrilateral formed by lines $AX$, $BY$, $CZ$, $O_AO_BO_C$.
Proof. It suffices to show that $O_AO_CYJ$ is cyclic as symmetry will finish. This can be done by noting \[\measuredangle O_CJO_A=\measuredangle JO_AO_C+\measuredangle O_AO_CJ =\measuredangle JAP+\measuredangle PCJ\]from the circumcenters. Now, from the quadrilateral, $\measuredangle JAP+\measuredangle PCJ=\measuredangle APC+\measuredangle CJA$. Furthermore, $\measuredangle CJA=\measuredangle CJP+\measuredangle PJA=\measuredangle FCC_0+\measuredangle A_0AD$ and adding this back to $\measuredangle APC=\measuredangle CHA$ we get $\measuredangle (CF,AD)$. Then $\measuredangle(O_CY,CF)=90^{\circ}=\measuredangle(O_AY,AD)$ so $\measuredangle(CF,AD)=\measuredangle(O_CY,O_A,Y)=\measuredangle O_CYO_A$ as desired. $\blacksquare$
Then this miquel point lies on $(XYZ)$ as desired.
This post has been edited 1 time. Last edited by GrantStar, Oct 1, 2023, 1:29 AM
Z K Y
The post below has been deleted. Click to close.
This post has been deleted. Click here to see post.
awesomeming327.
1677 posts
#21
Y by
https://media.discordapp.net/attachments/986049049834713139/1187418651440197702/Screenshot_2023-12-21_at_10.37.40.png

Let $\omega_A$, $\omega_B$, $\omega_C$ by $(ADP)$, $(BEP)$, $(CFP)$. Note that since the $A$-midline is a diameter of $\omega_A$, the reflection of $A$ over the $A$-midline is on $\omega_A$. Indeed, the foot of altitude from $A$ is on $\omega_A$. Similarly, the feet of altitudes from $B$ and $C$ are on $\omega_B$ and $\omega_C$, respectively. Now, let $H$ be orthocenter, and $H_A$, $H_B$, $H_C$ be the feet of altitudes from $A$, $B$, and $C$ respectively. Then $HA\cdot HH_A=HB\cdot HH_B=HC\cdot HH_C$, so \[\text{Pow}_{\omega_A}(H)=\text{Pow}_{\omega_B}(H)=\text{Pow}_{\omega_C}(H)\]which implies $\omega_A$, $\omega_B$, and $\omega_C$ are coaxial. Let $Q\neq P$ be their concurrency point.

$~$
Let the perpendicular bisector of $AD$ intersect the perpendicular bisectors of $BE$ and $CF$ at $Z$ and $Y$ respectively. Let the perpendicular bisectors of $BE$ and $CF$ intersect at $X$. Note that
\begin{align*}
\measuredangle QO_AY &= \measuredangle QO_AA - \measuredangle YO_AA \\
&= 2 \measuredangle QPA - 90^\circ - \measuredangle DAO_A \\
&= 2 \measuredangle HPA - \measuredangle DPA \\
&= 2 \measuredangle HPA - \measuredangle HAP \\
\end{align*}Similarly, $\angle QO_CY = 2\measuredangle HPC - \measuredangle HCP$. The rest is trivial triangle and circumcircle angle chasing.

$~$
Hence, $Q$ is on $(O_CO_AY)$. Similarly, it is on $(O_BO_CX)$ and $(O_AO_BZ)$ so by Miquel point, $Q$ is on $(XYZ)$ as desired.
This post has been edited 1 time. Last edited by awesomeming327., Dec 21, 2023, 3:39 PM
Z K Y
The post below has been deleted. Click to close.
This post has been deleted. Click here to see post.
thdnder
194 posts
#22 • 1 Y
Y by TheHU-1729
Let $l_A$ be the perpendicular bisector of $AD$ and define $l_B, l_C$ analogously. Let $X = l_B \cap l_C, Y = l_A \cap l_C, Z = l_A \cap l_B$.

Claim: $\omega_A, \omega_B, \omega_C$ concur at point $Q$ other than $P$.

Proof. Let $AA_1, BB_1, CC_1$ be the altitudes of $\triangle ABC$ and let $H$ be the orthocenter of $\triangle ABC$. Note that $A_1 \in \omega_A$, so the power of $H$ wrt $\omega_A$ is $AH \cdot HA_1$. Similarly the power of $H$ wrt $\omega_B, \omega_C$ are $BH \cdot HB_1, CH \cdot HC_1$. Hence $HP$ is the radical axis of $\omega_A, \omega_B, \omega_C$, so we're done. $\blacksquare$

By Simson's theorem, it suffices to show that to prove that the reflections $Q_A, Q_B, Q_C$ of Q in lines $l_A, l_B, l_C$, respectively, are collinear. We'll prove that $P, Q_A, Q_B, Q_C$ are collinear, completing the proof. Thus we only need to consider the following claim:

Claim: $P, Q_A, Q_B$ are collinear.

Proof. We'll prove that $\measuredangle(PQ_A, PQ) = \measuredangle(PQ_B, PQ)$. Note that $\measuredangle(PQ_A, PQ) = \measuredangle(PQ_A, PA) + \measuredangle(PA, PQ) = \measuredangle(DQ_A, DA) + \measuredangle(PA, PQ) = \measuredangle(Q_AD, Q_AQ) + \measuredangle(PA, PQ) = \measuredangle(PD, PQ) + \measuredangle(PA, PQ) = \measuredangle(BC, PQ) + 90^{\circ} + \measuredangle(PA, PQ)$. Similarly $\measuredangle(PQ_B, PQ) = \measuredangle(AC, PQ) + 90^{\circ} + \measuredangle(PB, PQ)$. So we only need to prove that $\measuredangle(BC, PQ) + \measuredangle(PA, PQ) = \measuredangle(AC, PQ) + \measuredangle(PB, PQ)$ and

$\measuredangle(BC, PQ) + \measuredangle(PA, PQ) = \measuredangle(BC, AC) + \measuredangle(AC, PQ) + \measuredangle(PA, PQ) = \measuredangle(BC, AC) + \measuredangle(AC, PQ) + \measuredangle(PA, PB) + \measuredangle(PB, PQ) = \measuredangle(AC, PQ) + \measuredangle(PB, PQ)$. $\blacksquare$

Therefore $P, Q_A, A_B$ are collinear and we're done. $\blacksquare$

Remark: This problem is very hard. Firstly I let $O_A, O_B, O_C$ be the center of $\omega_A, \omega_B, \omega_C$, respectively and trying to show $Q$ is the miquel point of the quadrilateral $O_BYZO_A$. But I couldn't prove that :P . Then I came with a nasty idea: Showing the projections of $Q$ onto the sidelines $l_A, l_B, l_C$ are collinear. That's how I came with the above solution. :-D
Z K Y
The post below has been deleted. Click to close.
This post has been deleted. Click here to see post.
math_comb01
662 posts
#23
Y by
Cool Problem
[asy] /* Geogebra to Asymptote conversion, documentation at artofproblemsolving.com/Wiki go to User:Azjps/geogebra */
import graph; size(12 cm); 
real labelscalefactor = 0.5; /* changes label-to-point distance */
pen dps = linewidth(0.7) + fontsize(10); defaultpen(dps); /* default pen style */ 
pen dotstyle = black; /* point style */ 
real xmin = -1.802134077018174, xmax = 9.623515320747276, ymin = -5.000351955319224, ymax = 4.235987796708523;  /* image dimensions */
pen zzttff = rgb(0.6,0.2,1); pen fuqqzz = rgb(0.9568627450980393,0,0.6); pen zzttqq = rgb(0.6,0.2,0); pen ffvvqq = rgb(1,0.3333333333333333,0); pen qqwuqq = rgb(0,0.39215686274509803,0); pen ccqqqq = rgb(0.8,0,0); 

draw((1.935599046713985,2.880503314568306)--(0.5091538630486546,-3.5685317315057876)--(9.223421273550613,-3.057722203594869)--cycle, linewidth(0.4) + zzttff); 
 /* draw figures */
draw((1.935599046713985,2.880503314568306)--(0.5091538630486546,-3.5685317315057876), linewidth(0.4) + zzttff); 
draw((0.5091538630486546,-3.5685317315057876)--(9.223421273550613,-3.057722203594869), linewidth(0.4) + zzttff); 
draw((9.223421273550613,-3.057722203594869)--(1.935599046713985,2.880503314568306), linewidth(0.4) + zzttff); 
draw(circle((4.73782473408456,-1.1215870128446004), 4.8856110970616), linewidth(0.4) + fuqqzz); 
draw((xmin, 0.058617609931882385*xmin-0.4156669946908912)--(xmax, 0.058617609931882385*xmax-0.4156669946908912), linewidth(0.4)); /* line */
draw((xmin, 4.521053539192397*xmin-25.313873600939353)--(xmax, 4.521053539192397*xmax-25.313873600939353), linewidth(0.4)); /* line */
draw((8.997963430632232,2.4033597355910414)--(9.223421273550613,-3.057722203594869), linewidth(0.4)); 
draw((0.5091538630486546,-3.5685317315057876)--(-2.1864201196666193,-3.404121069972541), linewidth(0.4)); 
draw((1.935599046713985,2.880503314568306)--(3.9773669748067504,-4.003819575642438), linewidth(0.4)); 
draw((xmin, 16.39537215881942*xmin + 10.26337584260156)--(xmax, 16.39537215881942*xmax + 10.26337584260156), linewidth(0.4)); /* line */
draw((xmin, 0.296582243548757*xmin-1.4384984948821595)--(xmax, 0.296582243548757*xmax-1.4384984948821595), linewidth(0.4)); /* line */
draw((xmin, 0.04128446440266929*xmin-0.7033112880955035)--(xmax, 0.04128446440266929*xmax-0.7033112880955035), linewidth(0.4)); /* line */
draw(circle((1.1312613320755578,-1.3041530719715546), 1.8908023089668118), linewidth(0.4) + blue); 
draw(circle((4.298250057771346,-0.16371384941475903), 3.8534890607784873), linewidth(0.4) + zzttqq); 
draw(circle((-0.558470376938768,1.1070461730142935), 4.79592016812677), linewidth(0.4) + ffvvqq); 
draw(circle((5.493712265514272,-0.47650631963137186), 4.535791521251047), linewidth(0.4) + qqwuqq); 
draw((3.9773669748067504,-4.003819575642438)--(3.530910495031583,3.6126028662479803), linewidth(0.4)); 
draw((3.530910495031583,3.6126028662479803)--(1.935599046713985,2.880503314568306), linewidth(0.4)); 
draw(circle((3.4651747246143474,-1.312081803843875), 2.442805548530799), linewidth(0.4) + ccqqqq); 
draw((1.2223764548813199,-0.34401420846874076)--(3.9773669748067504,-4.003819575642438), linewidth(0.4)); 
draw((1.2223764548813199,-0.34401420846874076)--(3.530910495031583,3.6126028662479803), linewidth(0.4)); 
draw((3.530910495031583,3.6126028662479803)--(5.579510160132299,-0.08860944451328145), linewidth(0.4)); 
draw((5.579510160132299,-0.08860944451328145)--(3.9773669748067504,-4.003819575642438), linewidth(0.4)); 
draw((1.935599046713985,2.880503314568306)--(2.3074470884511924,-3.4631200806760027), linewidth(0.4) + blue); 
draw((0.5091538630486546,-3.5685317315057876)--(4.236380809678523,1.0057922484490547), linewidth(0.4) + blue); 
draw((1.023319121598678,-1.2439630696084243)--(9.223421273550613,-3.057722203594869), linewidth(0.4) + blue); 
draw((1.7797181488137082,-3.080282889417355)--(3.530910495031583,3.6126028662479803), linewidth(0.4) + ccqqqq); 
draw((-0.558470376938768,1.1070461730142935)--(5.493712265514272,-0.47650631963137186), linewidth(0.4) + fuqqzz); 
draw(circle((2.386435790374484,0.00498821413650977), 3.144360678898891), linewidth(0.8) + green); 
 /* dots and labels */
dot((1.935599046713985,2.880503314568306),dotstyle); 
label("$A$", (1.9916476191987738,3.008035112912926), NE * labelscalefactor); 
dot((0.5091538630486546,-3.5685317315057876),dotstyle); 
label("$B$", (0.5634852586973722,-3.438716477013955), NE * labelscalefactor); 
dot((9.223421273550613,-3.057722203594869),dotstyle); 
label("$C$", (9.279279851290037,-2.918171317578865), NE * labelscalefactor); 
dot((1.2223764548813199,-0.34401420846874076),linewidth(4pt) + dotstyle); 
label("$R$", (1.2708927830578796,-0.23536164972109455), NE * labelscalefactor); 
dot((4.866287568299634,-3.3131269675503283),linewidth(4pt) + dotstyle); 
label("$M$", (4.914708899103511,-3.2118121767473773), NE * labelscalefactor); 
dot((5.579510160132299,-0.08860944451328145),linewidth(4pt) + dotstyle); 
label("$N$", (5.6354637352444055,0.018237274106256862), NE * labelscalefactor); 
dot((3.530910495031583,3.6126028662479803),dotstyle); 
label("$P$", (3.5799777210648185,3.7421372608342067), NE * labelscalefactor); 
dot((3.9773669748067504,-4.003819575642438),dotstyle); 
label("$D$", (4.033786321597974,-3.865830453986336), NE * labelscalefactor); 
dot((-2.1864201196666193,-3.404121069972541),dotstyle); 
label("$E$", (-2.1326717209407873,-3.2652014238689246), NE * labelscalefactor); 
dot((8.997963430632232,2.4033597355910414),dotstyle); 
label("$F$", (9.05237555102346,2.540879200599384), NE * labelscalefactor); 
dot((-0.6705777378484441,-0.7309957308429301),linewidth(4pt) + dotstyle); 
label("$G$", (-0.6110781779766775,-0.6224336913523152), NE * labelscalefactor); 
dot((-0.7268791256406042,-1.6540779367534089),linewidth(4pt) + dotstyle); 
label("$H$", (-0.6778147368786122,-1.5433982041990124), NE * labelscalefactor); 
dot((2.8797242547337776,-0.5844234146114434),linewidth(4pt) + dotstyle); 
label("$I$", (2.9393067556062458,-0.47561326176805907), NE * labelscalefactor); 
dot((1.7797181488137082,-3.080282889417355),linewidth(4pt) + dotstyle); 
label("$J$", (1.911563748516452,-3.1717702414062163), NE * labelscalefactor); 
dot((1.023319121598678,-1.2439630696084243),linewidth(4pt) + dotstyle); 
label("$K$", (1.0706831063520756,-1.142978850787405), NE * labelscalefactor); 
dot((2.3074470884511924,-3.4631200806760027),linewidth(4pt) + dotstyle); 
label("$L$", (2.3653723490496077,-3.358632606331633), NE * labelscalefactor); 
dot((4.236380809678523,1.0057922484490547),linewidth(4pt) + dotstyle); 
label("$O$", (4.287385245425326,1.112716840097984), NE * labelscalefactor); 
dot((2.1925247151441445,-1.502576594843139),linewidth(4pt) + dotstyle); 
label("$Q$", (2.2452465430261253,-1.3965777746147563), NE * labelscalefactor); 
dot((4.298250057771346,-0.16371384941475903),linewidth(4pt) + dotstyle); 
label("$S$", (4.35412180432726,-0.06184659657606464), NE * labelscalefactor); 
dot((-0.558470376938768,1.1070461730142935),linewidth(4pt) + dotstyle); 
label("$T$", (-0.5042996837335821,1.2194953343410795), NE * labelscalefactor); 
dot((5.493712265514272,-0.47650631963137186),linewidth(4pt) + dotstyle); 
label("$U$", (5.542032552781697,-0.36883476752496375), NE * labelscalefactor); 
clip((xmin,ymin)--(xmin,ymax)--(xmax,ymax)--(xmax,ymin)--cycle); 
 /* end of picture */ [/asy]
Let $\gamma$ be the nine point circle
Let $L,O,K$ be the foot of altitudes
Let $M,N,R$ be the midpoints
Let $S,T,U$ be the centers.
Let $Q$ be the orthocenter
Let $G,H,I$ be the intersections of perpendicular bisectors
Let $J = \gamma \cap QP$
Claim 1: $\gamma,(ADP),(BEP),(CFP)$ concurr
Proof
Then notice $S-T-U$
Claim 2: $J$ is the miquel point of $TGISUH$
Proof
Remark
Z K Y
The post below has been deleted. Click to close.
This post has been deleted. Click here to see post.
WLOGQED1729
38 posts
#24
Y by
Nice problem! :coolspeak:
We divide the problem into two parts.
Part(i):We’ll show that $(ADP),(BEP),(CFP)$ have a common point $T\neq P$
Define $A_1,B_1,C_1$ to be foot of altitudes from $A,B,C$ to its opposite sides and denote $H$ by the orthocenter of $ABC$
Note that $(ADP)=(APA_1),(BEP)=(BPB_1),(CFP)=(CPC_1)$
Observe that $Pow(H,(APA_1))=AH \cdot HA_1=BH\cdot HB_1=Pow(H,(BPB_1))=CH\cdot HC_1=Pow(H,(CPC_1))$ which means H has equal power wrt. these three circles. We have that $P$is the common point of these three circles, they must be coaxial circles which has $HP$ as the common radical axis. Thus, $(ADP),(BEP),(CFP)$ have a common point $T\neq P$ s.t. $T\in HP$. $\square$
_______________________________________________________________________________________________________________
Let $l_1,l_2,l_3$ the perpendicular bisectors of $AD,BE,CF$, respectively.
Define $A’=l_2\cap l_3, B’=l_3\cap l_1, C’=l_1\cap l_2$
Define $O_a,O_b,O_c$ be the centers of $(ADP),(BEP),(CFP)$, respectively.
By part(i) it is obvious that $O_a,O_b,O_c$ are collinear.
Observe that $O_a\in B’C’$, $O_b\in C’A’$ and $O_c\in A’B’$.
_______________________________________________________________________________________________________________
Part(ii): We’ll show that $(A’B’C’)$ pass through T
First, we prove the following claim.
Claim: $T\in (O_aO_cB’)$(Similary, $T\in(O_cO_bA’),(O_bO_aC’)$)
Proof: We proceed by angle chasing
$\measuredangle{O_aB’O_c}=\measuredangle{(l_1,AA_1)}+\measuredangle{AHC}+\measuredangle{(CC_1,l_3)}=90^\circ-\measuredangle{A_1AD}+\measuredangle{CBA}+90^\circ-\measuredangle{FCC_1}=\measuredangle{DAA_1}+\measuredangle{CBA}+\measuredangle{C_1CF}=\measuredangle{AA_1P}+\measuredangle{CBA}+\measuredangle{PC_1C}=\measuredangle{ATP}+\measuredangle{TPA}+\measuredangle{CPT}+\measuredangle{PTC}=\measuredangle{TAP}+\measuredangle{PCT}=90^\circ+\measuredangle{TAP}+90^\circ+\measuredangle{PCT}=\measuredangle{O_aTP}+\measuredangle{PTO_c}=\measuredangle{O_aTO_c}  \square$

By the claim we can conclude that $T$ is the Miquel point formed by {$l_1,l_2,l_3,\overline{O_aO_bO_c}$}
So, $T\in(A’B’C’)$, as desired $\blacksquare$
_______________________________________________________________________________________________________________
Remark : First, it is natural to consider only $(ADP),(BEP),(CFP)$ which we can view those three circle as $(APA_1),(BPB_1),(CPC_1)$, respectively. Then we just consider power of $H$ wrt. those three circles. Finally, The key step is to claim that the intersection point is the Miquel point which has the same idea as IMO 2011/6 (In 2011/6 the tangency point is the Miquel point of complete quadrilateral formed by lines $l_a,l_b,l_c$ and the Steiner line of the tangency point)
Final Remark : When you see a problem which introduce a triangle forms by three lines it is a good idea to consider Miquel point of some complete quadrilateral.
Attachments:
This post has been edited 2 times. Last edited by WLOGQED1729, May 29, 2024, 9:22 AM
Reason: .
Z K Y
The post below has been deleted. Click to close.
This post has been deleted. Click here to see post.
Scilyse
386 posts
#25
Y by
Longest angle chase I've ever done :wacko:

Let $\Gamma$ be the circumcircle of $\triangle ABC$, and let $\gamma$ be its nine-point circle. Further let $H_A$, $H_B$, $H_C$ be the feet of the perpendiculars from each vertex to the opposite side, and let $H$ be the orthocenter of $\triangle ABC$. Let $A'$, $B'$, $C'$, and $P'$ be the midpoints of $\overline{AH}$, $\overline{BH}$, $\overline{CH}$, and $\overline{PH}$, which all lie on $\gamma$ by a homothety at $H$. Let $Q = HP' \cap \gamma \neq P'$, and let $\ell_A$, $\ell_B$, and $\ell_C$ be the three midlines of $\triangle ABC$ parallel to $BC$, $CA$, and $AB$, respectively.
Note that $\ell_A$, $\ell_B$, and $\ell_C$ are the perpendicular bisectors of segments $\overline{AH_A}$, $\overline{BH_B}$, and $\overline{CH_C}$, respectively. Therefore $ADPH_A$, $BEPH_B$, and $CFPH_C$ are (possibly self-intersecting) isosceles trapeziums, which implies that they are cyclic.

Claim: $(ADP)$, $(BEP)$, and $(CFP)$ pass through $Q$.
Proof. Note \[HA \cdot HH_A = 2 HA' \cdot HH_A = 2 HP' \cdot HQ = HP \cdot HQ,\]and cyclic variations thereof also hold. $\blacksquare$
Now let $m$, $m_A$, $m_B$, and $m_C$ be the perpendicular bisectors of segments $\overline{PQ}$, $\overline{AD}$, $\overline{BE}$, and $\overline{CF}$. Let $X'$, $Y'$, and $Z'$ be the circumcenters of cyclic pentagons $ADPH_AQ$, $BEPH_BQ$, and $CFPH_CQ$. Clearly $X'$ is the intersection of the perpendicular bisectors of $\overline{AD}$, $\overline{AH_A}$, and $\overline{PQ}$, and therefore $\ell_A$, $m_A$, and $m$ concur at $X'$. Similarly, $\ell_B$, $m_B$, and $m$ concur at $Y'$, and $\ell_C$, $m_C$, and $m$ concur at $Z'$.
Now we contend the following, which solves the problem:
Claim: $Q$ is the Miquel point of the complete quadrilateral formed by lines $m$, $m_A$, $m_B$, and $m_C$.
Proof. By symmetry it suffices to prove that $Q$ lies on the circumcircle of the triangle determined by lines $m$, $m_B$, and $m_C$, which is $(XY'Z')$. Now \begin{align*} \measuredangle Y'XZ' &= \measuredangle(m_B, m_C) \\ &= - \measuredangle(CF, BE) \\ &= \measuredangle(BE, CF) \\ &= \measuredangle(BE, \ell_B) + \measuredangle(\ell_B, \ell_C) + \measuredangle(\ell_C, CF) \\ &= \measuredangle(\ell_B, H_B P) + \measuredangle CAB + \measuredangle (H_C P, \ell_C) \text{ (reflection over }\ell_B\text{ and }\ell_C\text{)}\\ &= \measuredangle H_C PH_B - \measuredangle(\ell_C, \ell_B) + \measuredangle CAB \\ &= \measuredangle H_C PH_B + 2 \measuredangle CAB. \end{align*}However, \begin{align*} \measuredangle Y'QZ' &= \measuredangle Y'QP + \measuredangle PQZ' \\ &= 90^\circ - \measuredangle PH_B Q + 90^\circ - \measuredangle QH_C P \\ &= \measuredangle QH_B P  + \measuredangle PH_C Q \\ &= - \measuredangle H_B PH_C  - \measuredangle H_C QH_B \text{ (angle sum in a quadrilateral)} \\ &= \measuredangle H_C PH_B + \measuredangle H_B QH_C \\ &= \measuredangle H_C PH_B + \measuredangle H_B A'H_C \\ &= \measuredangle H_C PH_B + 2 \measuredangle H_B AH_C \\ &= \measuredangle H_C PH_B + 2 \measuredangle CAB, \end{align*}as $A'$ is the circumcenter of $\triangle AH_B H_C$. $\blacksquare$
Z K Y
The post below has been deleted. Click to close.
This post has been deleted. Click here to see post.
Ywgh1
138 posts
#26
Y by
2020 G7
[asy]
 /* Geogebra to Asymptote conversion, documentation at artofproblemsolving.com/Wiki go to User:Azjps/geogebra */
import graph; size(11.92465773392315cm); 
real labelscalefactor = 0.5; /* changes label-to-point distance */
pen dps = linewidth(0.7) + fontsize(10); defaultpen(dps); /* default pen style */ 
pen dotstyle = black; /* point style */ 
real xmin = -6.728037610265662, xmax = 34.19662012365749, ymin = -12.973956614346903, ymax = 6.454288251680905;  /* image dimensions */
pen qqffff = rgb(0.,1.,1.); pen ffdxqq = rgb(1.,0.8431372549019608,0.); pen zzttqq = rgb(0.6,0.2,0.); pen ffxfqq = rgb(1.,0.4980392156862745,0.); pen qqwuqq = rgb(0.,0.39215686274509803,0.); pen bfffqq = rgb(0.7490196078431373,1.,0.); pen yqqqqq = rgb(0.5019607843137255,0.,0.); pen eqeqeq = rgb(0.8784313725490196,0.8784313725490196,0.8784313725490196); pen dcrutc = rgb(0.8627450980392157,0.0784313725490196,0.23529411764705882); 
 /* draw figures */
draw(circle((8.7,-0.62), 6.489219022592791), linewidth(1.) + qqffff); 
draw((3.0477872806906086,-3.807860551977967)--(14.359595892000863,-3.794734297925905), linewidth(1.) + ffdxqq); 
draw((14.359595892000863,-3.794734297925905)--(6.764031378993902,5.573705596947589), linewidth(1.) + ffdxqq); 
draw((6.764031378993902,5.573705596947589)--(3.0477872806906086,-3.807860551977967), linewidth(1.) + ffdxqq); 
draw((4.905909329842255,0.8829225224848107)--(10.561813635497384,0.8894856495108419), linewidth(1.) + zzttqq); 
draw((10.561813635497384,0.8894856495108419)--(8.703691586345736,-3.801297424951936), linewidth(1.) + zzttqq); 
draw((8.703691586345736,-3.801297424951936)--(4.905909329842255,0.8829225224848107), linewidth(1.) + zzttqq); 
draw((6.764031378993902,5.573705596947589)--(15.155068428396824,1.7646617462628544), linewidth(1.) + red); 
draw((3.0477872806906086,-3.807860551977967)--(3.6258089506720923,-9.324137255212241), linewidth(1.) + red); 
draw((14.359595892000863,-3.794734297925905)--(6.621046609852666,3.406284815596815), linewidth(1.) + red); 
draw(circle((5.301172191355441,-3.6919644189204277), 2.798667710105094), linewidth(1.) + ffxfqq); 
draw(circle((9.697272918493903,0.8884824340676138), 5.52767781396812), linewidth(1.) + dotted + qqwuqq); 
draw(circle((10.349441616289988,-5.831180814279533), 7.576805456154811), linewidth(1.) + dotted + qqwuqq); 
draw(circle((9.867330712887176,-0.86371928144121), 5.3638880922644265), linewidth(1.) + dotted + qqwuqq); 
draw((4.693418138060977,-6.423845815007937)--(6.462026217482862,-6.238522661256276), linewidth(1.) + bfffqq); 
draw((6.462026217482862,-6.238522661256276)--(10.959549903695363,3.669183671605223), linewidth(1.) + bfffqq); 
draw((4.693418138060977,-6.423845815007937)--(10.490321250926764,-0.1942247411645448), linewidth(1.) + bfffqq); 
draw((6.462026217482862,-6.238522661256276)--(10.349441616289988,-5.831180814279533), linewidth(1.) + bfffqq); 
draw((8.703691586345736,-3.801297424951936)--(10.349441616289988,-5.831180814279533), linewidth(1.) + yqqqqq); 
draw((3.3367981156813507,-6.565998903595104)--(4.693418138060977,-6.423845815007937), linewidth(1.) + bfffqq); 
draw((10.349441616289988,-5.831180814279533)--(9.697272918493903,0.8884824340676138), linewidth(1.) + eqeqeq); 
draw((15.157087169260315,0.02497194648601364)--(15.155068428396824,1.7646617462628544), linewidth(1.) + dcrutc); 
draw(circle((7.735707275842686,-0.7044446264781421), 3.244609511296395), linewidth(1.) + green); 
 /* dots and labels */
dot((3.0477872806906086,-3.807860551977967),dotstyle); 
label("$B$", (3.174100095645299,-3.4791853330462166), NE * labelscalefactor); 
dot((14.359595892000863,-3.794734297925905),dotstyle); 
label("$C$", (14.486352348284084,-3.4791853330462166), NE * labelscalefactor); 
dot((6.764031378993902,5.573705596947589),dotstyle); 
label("$A$", (6.0570009467333,5.702227160092731), NE * labelscalefactor); 
dot((15.157087169260315,0.02497194648601364),dotstyle); 
label("$P$", (15.269749318688433,0.34379188252699716), NE * labelscalefactor); 
dot((4.905909329842255,0.8829225224848107),linewidth(4.pt) + dotstyle); 
label("$M_c$", (5.022916945799561,1.1271888529313443), NE * labelscalefactor); 
dot((10.561813635497384,0.8894856495108419),linewidth(4.pt) + dotstyle); 
label("$M_b$", (10.69471101152704,1.1271888529313443), NE * labelscalefactor); 
dot((8.703691586345736,-3.801297424951936),linewidth(4.pt) + dotstyle); 
label("$M_a$", (8.814558282556604,-3.5418570906785645), NE * labelscalefactor); 
dot((3.6258089506720923,-9.324137255212241),dotstyle); 
label("$E$", (3.7381459143364295,-9.025635883508993), NE * labelscalefactor); 
dot((15.155068428396824,1.7646617462628544),dotstyle); 
label("$D$", (15.269749318688433,2.0672652174165607), NE * labelscalefactor); 
dot((6.621046609852666,3.406284815596815),dotstyle); 
label("$F$", (6.746390280689126,3.7280667946737767), NE * labelscalefactor); 
dot((6.462026217482862,-6.238522661256276),linewidth(4.pt) + dotstyle); 
label("$I$", (6.589710886608257,-5.986055638340127), NE * labelscalefactor); 
dot((7.982311640122614,-2.8894482459685378),linewidth(4.pt) + dotstyle); 
label("$J$", (8.093833069784605,-2.633116605009522), NE * labelscalefactor); 
dot((4.693418138060977,-6.423845815007937),linewidth(4.pt) + dotstyle); 
label("$K$", (4.803565794086343,-6.1740709112371706), NE * labelscalefactor); 
dot((4.505405406686321,-1.0088139989845155),linewidth(4.pt) + dotstyle); 
label("$Q$", (4.6155505211892995,-0.7529638760390888), NE * labelscalefactor); 
dot((10.349441616289988,-5.831180814279533),linewidth(4.pt) + dotstyle); 
label("$O_3$", (10.475359859813823,-5.5786892137298665), NE * labelscalefactor); 
dot((9.867330712887176,-0.86371928144121),linewidth(4.pt) + dotstyle); 
label("$O_{2}$", (10.005321677571214,-0.6276203607743932), NE * labelscalefactor); 
dot((9.697272918493903,0.8884824340676138),linewidth(4.pt) + dotstyle); 
label("$O_1$", (9.81730640467417,1.1271888529313443), NE * labelscalefactor); 
clip((xmin,ymin)--(xmin,ymax)--(xmax,ymax)--(xmax,ymin)--cycle); 
 /* end of picture */
[/asy]
Here is a sketch of the solution.

Let $\omega_a, \omega_b$ and $\omega_c$ denote circles $\triangle ADP, \triangle BEP, \triangle CFP$, let $H_1,H_2.H_3$ denote the $A,B,C$ feet of altitude, and let $H$ denote the orthocenter of $\triangle ABC$. Firstly, we want to show that $\omega_a, \omega_b$ and $\omega_c$ concur at the same point.
We claim that the $N_9$ circle pass through that point.
It's easy to show that $H_1 \in \omega_a$ similarly for $H_2,H_3$.
Now by power of a point we get that $HH_1\cdot HA= HH_2 \cdot HB = HH_3 \cdot HC$ hence $\omega_a, \omega_b$ and $\omega_c$ concur at the same point, call it $Q$.

Claim: We claim that $Q$ is the miquel point of $JIO_2O_3$.
Proof: Huge angle chase $\blacksquare$

This implies that $Q \in \omega $ hence we are done.
Z K Y
N Quick Reply
G
H
=
a